You are on page 1of 79
lel) abil Ail By YANN Ll Bygall — ale Gb a guS gl 9S cililaia) Als GL alas AMG SLE YL asi ALLY gle ALS YL call Lub dada obs bel 6 et pall ele Ganadall Sa pi dandy Ais ples pans GaiaYl oy 5 Aged g daa GUS =) (Ost alt) els Fase Aaa gle Gig ald Galea yt lf Jai! a1 we pl gl lant] eae AdaYi aie Gala Il Alls: pateall oyallede — -f glee) poh yall igs 0 AINEY gai Saal Ula} 4 SI Gary Ue ISA 2385 OF 8 Gl ol gall Maye Gal WUE GyLa Gs eLgii) ke 295 Gh ple Sa pe MAMI A yy antes) Sey Unt gig Sal A wey! Shas pall dala gS WAI Ay gle UES pre 8 Gj YE AGL; Spa) yy gle Maga dita 1+ MeL glade san 21) IVa As Gave dia Se BY neal UI GY Gas ab uae lt pe sl gl sb salle slee ge US volie jy sled opal Gass |e Ministry of Education and Higher Learning Medicine Colloquium Exam - First Session 2012 IMPORTANT INSTRUCTIONS Each candidate should read this page before answering the questions. The candidate should follow the directions below : L 2 3. 4- 5. 6- 7- 8 9. 10- dl. 12- Write clearly your FIRST and FAMILY NAME and APPLICANT NUMBER with ink (pen) in the designated space (colored square) Please use the pencil ONLY. Do not use any pen (like bic) or ink. Completely Fill the answer squares using pencil only. Only ONE answer is allowed to each question, Avoid erasing as much as you can. When needed, do erase the cancelled answer COMPLETELY. You may use the question bookle as scratch, and make sure to return it with the answer sheet at the end of the exam. On the answer sheet, you should fill up the square corresponding to the right answer, in pencil only. No writing allowed at all. Keep your answer sheet clean and tidy, do not fold or tear it. The duration of the exam is THREE HOURS. The composition consists of 180 QUESTIONS. Important Note : ‘Any square filled up with a pen or ink cannot be read by the machine. It reads answers filled up in PENCIL only. Thank You & Good Luck Ministere de Education et de l'Enseignement Supérieur Examen Colloquium Medecine ~ Premiere Session 2012 INSTRUCTIONS IMPORTANTES Le Candidat doit lire ces instructions avant de commencer a répondre aux questions et devra observer les directives suivantes Ecrire son PRENOM et NOM et son NUMERO D'EXAMEN & Tencre et de facon lisible a l'endroit indiqué (coin en couleur) Utiliser UNIQUEMENT un crayon mine. Ne pas utiliser de stylo a encre ou 4 bille (encre séche). Remplir, sur la fiche de réponse, tout le carré au complet, sans sortir de lignes. Chaque question qui comporte PLUS D'UNE réponse, est annulée. Essayer d'éviter, autant que possible, d'effacer. Dans des cas particulier, il faut TRES BIEN effacer la réponse que vous voulez changer. Pour éviter les erreurs, vous pouvez utiliser la feuille de questions comme brouillon, bien que vous devez la rendre, 4 la fin de 'épreuve. Interdiction formelle d’écrire quoique ce soit sur la feuille de réponse en dehors des carrés. Bien conserver la feuille de réponse ; elle ne doit pas étre froissée. La durée de l'examen est de TROIS HEURES. La composition comprend 180 QUESTIONS. Remarqu Chaque carré rempli d'encre ou de toute couleur autre que la mine, ne sera pas compté car la machine qui lit les réponses ne peut capter que le signal envoyé par le carré rempli de MINE. Merci & Bonne Chance ‘The most appropriate test to screen for late-onset male hypogonadism is A) free testosterone B) total testosterone LH D) FSH Le test de dépistage le plus approprié pour I"hypogonadisme masculin tardif est : A) la testostérone libre B) la testostérone totale C)LH D) la FSH LAD pg Gah AAA SSVI Gl OD PS ae La cls all 2 oy a sal “A, AS Gy pe fal -B LH -C FSH -D ‘Which one of the following is contiaindicated in the second ‘and third trimesters of pregnancy? COG, GAD Daal JS Casha ae ya Gag SSatho A) Amoxicillin os o B) Azithromycin (Zithromax) Glee A ©) Ceftriaxone (Rocephin) ora 3) -B ) Ciprofloxacin (Cipro) mth -C E) Doxyeyctine sells shy sane -D Lequel des medicaments suivants, est contre-indiqué dans le oleae EB deuxieme et troisiéme trimestre de la grossesse? A) Amoxieilline B) Liazitlromycine (Zithromax) ©) La ceftriaxone (Rocephin) D) La ciprofloxacine (Cipro) B) La doxyeyoline 3-_| Allthe followings are risk factors for asthma except one: We Uy hs De a bs A. Atopic decmattis “Asad gal A B. Repeated bronchiolitis “ , C. Prematurity 2S Slama ll -B D. Hypereosinophilia ist E, Male gender civeda 4) -D SB Gal Toutes les propositions suivantes sont des facteurs de risque dasthme, saur: ‘A. La dermatite atopique B. Broncbiolite a répétition C. La prématurité D. Hyperéosinophilie E. Sexe male [4] A 30-year-old previously bealthy man complains of he Shop tans lae Fone day chronic, intensely pruritic vesicular rash He gives a I-year as ah Si : im history of frequen! abdominal pain, nonbloody ciamhea, and. | itt M28 Se SHAS Onde gS ght a 10 kg weight loss. He has no history of travel, antibiatic 1s SY Sh Ah ate See Daa «i i gl ol Oye use, of consumption of well wate Bee a hepa ene phy ly shad 55 The mos likey case his stoi peal AY bes SSVI Ca el actese intolerance nay B) irritable bowel syadrome _ DARN dese “A C) collagenous colitis ees gla -B D) celiac disease aS asf cteal -C E) Crohn's disease duels -D ‘Un homme de 30 ans aupatavant en bonne santé se plaint arSee E |_| dune éruption prurigineuse vésiculaize intense et chronique. || donne une histoire de I an de douleurs abdomtinales | fréquentes, de la diarthée non sanglante et une perte de 10 kg. dé poids. I n'a pas dantécédents de voyage, d'utilisation entibiotiques, ou consommation de Veau de puits. La cause la plus probable de ses symptdmes est AA) Fintolérance au lactose B) Le syndrome du c6lon irtable ©) a colite collagene D) la maladie coctiaque ) La maladie de Crohn 3 | Allthe following are ertical features in accidental dug poisoning except one: ‘A. Decreased consciousness and respiration B. Convalsions ©. Cardiac arrhythmia D. Hyperthermia E. Vomiting “Tout ce qui suit sont des caractéristiques essencelles d'une intoxication médicamenteuse accidentele, sauf une: ‘A. Diminution de le conscience et de la respiration B. Convulsions C. Anjthmie cardiaque D. Hyperthermie E, Vomissements IDM pan ADS, Gi Ral alg a Gg Le US teen! al ea hats OA os! -B oi ius YC gusabs -D lil -E | Patients with rheumatoid arthritis should be screened for tuberculosis before starting which one of the following medications? A) Gold B) Hydroxychloroquine ©) Infliximab D) Methotrexate &) Sulfasalazine Les patients attents de polyarthrte thumatoide doivent étre dépistés pour la tuberculose avant de commencer lequel des medicaments suivants? A) Or B} Ltaydroxychloroquine © Linfiiximad D) Le methotrexate ) La sulfasalazine CA Ge asia gen Se all gaa el a Ga TASK 91 Ge gly eal Jag 9 eae uA, O23 a B esi -C cbs -D ool. BE, ‘Methotrexate given intrathecally causes neurotoxicity acutely manifested as which of the following syndromes? A. Ototoxicity B. Axonal peripheral neuropathy C. Myopathy D. Aseptic meningitis E. Encephalopathy La méthotrexate administrée par voie intrathécale cause une eurotoxicité aigué manifestée par lequel des syndromes suivants? A, ot0toxicité B. neuropathie périphérique axonale C. myopathie D. La méningite aseptique E.Eneéphalopathie Be han cn al JSS CLAS Gg eel Sh SADA ce Dhl Oa hyd volt Raed eel OA ome ghene gueae ae -B ae Biel -C ill Yael gt -D bles Del -E The breastfed infant of a mother who isa strict vegan may experience deficiency of which of the following vitamins if the mother is not receiving supplements of the vitamin? AK B.B6 c.BI2 D. Folate E, Biotin Le nouveau-né allaité dune mere qui est une végétarienne striote peut éprouver un déficit vitaminique si la mére ne regoit pas de suppléments de Ja vitamine? AK B.B6 c.BI2 D. folate E. biotine ig pn By aye ny il al G iS 6 ANS Chagall Oo high Se Stans onal Lin ga Chae JY ally ‘The ability to walk along a straight line touching the heel of one foot fo the toe of the other is most often impaired with A. Cerebellar dysfunction B. Parietal lobe damage C. Temporal lobe damage D. Ocular motor disturbances E, Dysesthesias in the feet La capacité de mercher le long d'une ligne droite en touchant Je talon d'un pied & Vortel de autre est le plus souvent altérée avec A. Dysfonction cérébelleuse B. Atteinte du lobe pariétal C. Atteinte du lobe temporal D. Perturbations occulomottices E, Dysesthesies dans les pieds ae yp BS ell ge 1 Clana a le go AN ll cal PAS Ad “A glial gall pal -B. seal pall Bal -C a Spa BU baal -D Gel J caine -E ial gh 10 ‘A 68-year-old woman with primary hypotliyroidismn is taking levothyroxine, 125 g/day. Her TSH ievel is 6.2 uU/mL. (N 0.5-5.0). She has no symptoms of either bypothyroidism or hyperthyroidism. Which one of the following would be most appropriate at this point? ‘A) Continuing fevothyroxine at the same dosage B) Increasing the levothyroxine dosage C) Decreasing the levothyroxine dosage D) Discontinuing levothyroxine E) Orderinga free T4 Une ferme de 68 ans avec hypothyroidie primaire prend de a levothyroxine, 125 ug / jour. Son taux de TSH est de 0,2 U/mL (N 0,5 & 5,0). Elle n'a pas de symptémes hypothyroidie ow hyperthyroicie, Lequel des éléments suivants serait le plus approprié a ce stade? A) Continuer la lévothyroxine a la méme posologie B) L’augmentation de la dose de levothyroxine ©) La diminution de la dose de lévothyroxine D) Liat de Ia levothyroxine E) La commande duun T4 libre PGE Gao AS Ue Ve yl TSH Shee pl QSa0 V8 QoS sib ay lel an ual (22662 gual JalgSe +60 SY) ap Mion gl Go dbob A jb gas Fda pin se De Bel a oS shld I LN A nS siuilie 9 34) -B mS si he ym pans -C oes all i -D ITS thet 5M EB ist Which one of the following statement about absolute contraindications to electrocanvulsive therapy (ECT) is, correct? A) Age >80 years B) A cardiac pacemaker C) An implantable cardioverter-defibnillator D) Pregnancy E) There are no absolute contraindications to ECT Laquelle des énoncés suivantes est vraic au sujet des contre- indications absolues a la thérapie par électrochoes? A) Un Age> 80 ans B) Un stimulateur cardiaque ©) Un défibrilateur & synchronisation automatique implantable D) Grossesse E) II n'y a pas de contre-indications absolues & l'electrochos Gunde eae yoe yt 5 phe wl galls Still Le As on ohne A sulbAab 8. spb cis Me I yl deme -C gel -D at lee tal shina loa Gul -E PheS glass A. Cardiac involvement B. Arthritis CC. Nephritis D. Pancreatitis E. Meningitis Chez les enfants la complication la plus frequente des oreillons est la suivante: A.Latteinte cardiaque B. L'artrite C. La népiante D. La paneréatite E. La méningite 12 A Se-year-old woman presents with a two day history oF (gle Sa plane go Se Ua pe pt severe headache that improves alittle when she shakes her : : head, Which ype of headache may she have? Nestea nt sy! ely se late omy ‘A. Common migraine Ch tiene “A B. Muscle tension yet B CC. Classic migraine SS piesglne -C D. Cluster wots gle -D EB, Hypertensive ‘an pls) -E Une femme de 36 ans se présente avec une histoice de maux de tte violents depuis deux jours qui s’arnéiiorent un pew quand elle secove la tee. Quel type de maux de tate peut-lle avoir? A. Migraine commune B. La tension musculaire C. Migraine classique D, Cluster E. Hypertension [13 [In children the most common complication of mumps is aah a GUL ae te gt SV Chicana gh sity abs =A deatia hath -B poet -C Sebel -D este -E ia ‘A 55-year-old hypertensive man develops sudden onset of excruciating pain beginning in the anterior chest, and then radiating to the back. Over the next 2 hours, the pain moves downward toward the abdomen. The most likely diagnosis, is: Aortic dissection Aortic valve stenosis Atherosclerotic aneurysm Myocardial infaretion Syphilitic aneurysm moom> Un homme de 55 ans hypertendu développe apparition soudaine d'une douleur atroce & partir de la paroi thorecique anterieure, puis irradiant dans le dos. Au cours des 2 prochaines heures, la douleur se déplace vers le bas vers abdomen, Le diagnostic le plos probable est: A. La dissection aortique B. sténose de la valve aortique C. Anévrisme athéroscléreux D, Linfarctus du myocarde E, Anévrisme syphilitique Wie dei LB ge gy Ue 00 eu Jay pe RS ALM yal Ata lay ap aa a gai MS gael ay 0 aoe 2 Gi 8 ISM om weigh A plea gest B edhe gael Syd dhe ols! DD Batted OE except one: A. Convulsions B. Hyperventilation C. Respiratory alkalosis D. Aciduria E. Dehydration Tous les propositions qui suivent sont caractéristiques de intoxication aux salicylés, sauf une; A, Convulsion B. Hyperventilation C. Alealose respiratoire D. Acidurie E, Déshydratation TS _| The intrinsic pathway factors of coagulation does NOT Td Gs A ale Ol» gM ng include which factor: Vides A A. Factor ViIL B. Factor IX Me B C. FactorX Xote ‘D, Factor XI XI date D E. Factor XII ‘XII dake E Les facteurs dela voie intrinséque de la coagulation ne comprennent pas A. Facteur Vill B. Facteur IX ©. Facteur X D. Facteur XI E. Facteur XI T6_| Allthe following are features of salicylates intoxication Tie Gena peal Tian ya gh os lg “A kwbs -B iain -C Apes dy -D Gis -E 17] You are asked o perform e preaperative evaluation ona 55- | wae dp galyall San OA pu pl din ll ‘year-old woman with type 2 diabetes mellitus prior to nm Ste oo elective femorat-anterior tibial artery bypass surgery. She is easily) beg Se chon fis te ‘unable to climb a flight of stairs or do heavy work around the cette cg pall Gs yt Alp g03 Jal ye da gad | | house. She denies exertional chest pain, and is otherwise | dan psi jl aDte came Sh OI gas gil | | heaithy wr eas Beton cument udetines, which on ofthe flowing | 2s Ae Hush caage e dm ses diagnostic studies would be appropriate prior to surgery cmd eee because the results could alter the management of this SAN gag ha Ge gl 999 ll SVD patient? el ge SI Seg Ql GW dal all SBD A) Paimonary function suis Fay pag ga Ra B) Coronary angiography pass | C) Carotid angiography ED Aas tal pA | D) A dipyridamole-thallium scan gis! phe glues -B E) Ahemoglobin Alc level Ghmagle yhshss -C Selig pay aes pa chez une femme de SSans disbétique de type2 ayant un cute pontage électif entre Fartére fémorale et Fartéretibiale antérieure, Elle est incapable de monter un escalier ou faire des travaux lourds autour de la maison. Elle nic ls douleur thorecique a effort, et est en bonne santé Selon les lignes directrices actuelles, laquelle des études diagnostiques suivantes, serait epproprié avant la chirurgie parce que les résultats pourraientaltérer la conduite a tenir chez cette patiente? A) Les études de la fonction pulmonaire B) Langiographie coronaire C) Angiographie carotidienne D) Une scintigraphic au thallium dipyridamole E) Un niveaud’hémoglobine Ale 18 A primigravida at 38 weeks gestation is concemed that her fetus is getting too large. Ultrasonography is performed and an estimated fetal weight of 4000 g is reported Which on¢ of the following management options is supported by the best evidence? A) Induction of labor B) Cesarean section ) Awaiting spontaneous labor D) Weekly ultrasonography to follow fetal growth Une primigravide & 38 semaines de gestation est inquitéte parceque son foetus augmente de taille. Une echographie est pratiquée et le poids foctal est estimé & 4000g. Quelle est la ‘meilleure conduite a tenir évidente induction du travail Césarienne Attendre le travail spontané Echographie hebdomadaite afin de suivre la croissance du foetus pop> Saja Gu CaaNA Eg gh ep IN Sale 59 O28 Spall BUN gal oi SS a ARID a gh Geet 9 Gill toa BY uaa “A Lnwibe -B ith aoy gh jal -C Stall yo tated be pal pe dd -D 19 “A 43-year-old man has @ blood pressure of 150/95 mm Hg, and creatinine 1.4mg/d. His urinalysis shows no glucose, blood, or protein, and no casts. If he is not treated, which of the following conditions will most likely cause his death? Intracerebral hemorrhage Aortic aneurysm rupture Chronic renal failure Congestive heart failure Intracranial aneurysm rupture PUOD> Un homme de 43 ans a une pression artérielle de 150/95 mm Hget une créatinine .4 mg/j. Son analyse durine ne ‘montre pas de glucose, du sang ou des protéines, ni de cylindres. S'il n'est pas traité, la cause de sa mort serait, A, Lhémerragic intracérébrale B. Rupture d'un anévrisme aortique C. Linsuffisance rénale chronique D. L’insuffisance cardiaque, congestive E. Rupture d'un anévrisme intracténien Gajepte/\es ahicn Ole Piao day sama ook dal ih aaa Vk gt Sl 2) SON ey Vyas IA DEE sonal G85 (Say A CL a gh gag reas pablus A eM 550 B ey Sars D B PERN yp atl Js gall 56 [20 Upper extremity deep-vein thrombosis is best treated with: A) Intravenous heparin for 72 hours, followed by oral warfarin (Coumadin) for 3 months B) Low molecular weight heparin (LMWH) subcutaneously for 5 days only C) LMWH subcutaneously for at last 5 days, followed by oral warfarin for 3 months D) LMWH subcutaneously for at least § days, followed by oral warfarin indefinitely E) Oral warfarin for 3 months La thrombose veineuse profonde de Vextrémité supérieure est mieux traitée avec: ‘A) Lihépatine intraveineuse pendant 72 heures, la warfarine par voie orale (Coumadin) pendant 3 roois B) Ihéparine de bas poids moléculaire (HBPM) sous-cutanée pendant 5 jours seulement C) HBPM sous-cutanée pendant au moins 5 jours, suivie par 1a warfarine orale pendant 3 mois 1D) HBPM sous-cutanée pendant au moins 5 jours, suivie par a warfarine orale indéfiniraent ) la warfarine orale pendant 3 mois GOEW gi Good gaya) KAD CH Gaal ot A Ral PS OUI yy del VT al ga Oe “A ed VP bad Saal alalh Gins ine ine O39 Gates -B pov coke stalh cans Gane glia 3s Ib ea stew -C eed bag yo 2 LA AY fa SAT Baad ala ans Gaia i O91 Gato -D sae ot gd ds Sb ge ol et Tad god ola -E far 'A youttg girl te wans(ened lo the emergency tom because of 2 suicide attempt. On examination, she was agitated with wheezing and had increased bronchial secretion, miosis and diaphoresis, In addition to ventilation and perfusion, how do you deal with this girl? ‘A. You install a scope and you give physostigmine B. You make a benzodiazepine dosage in plasma then you give Flumazenil C. You give of atropine and pralidoxime D. You alealinise urine with serum bicarbonate E, You give Narcan Une jeune fille est wansférée aux urgences pour supposition d'une tentative de suicide. A l'examen, elle a une agitation avec une respiration bruyante par encombrement bronchique, tun myosis et une hypersudation. En plus de la ventilation et de la perfusion, comment traitez-vous cette fille? A. Vous installez un scope et vous tui donner la physostigmine B. Vous faites un dosage des benzodiazépines plasmatiques et vous donner le Flumazénil ©. Vous iui donnez de latropine et de la pralidoxime D, Vous alealinisez les urines avec di sérum bicarbonaié E. Vous donnez le Narcan JR a a AE Gl I ah haa Jadt ee cl A CaS «peal Ce RY WF Geis than Geet dame LAY! Sa ak a Ua pa HS yy Gishsnty A gles jhe UR “A apes SL DW ow Jha see hed -B el eS sil cattle -C bas Shi Sy othels Jott cuts -D Sys -E 22 While evaluating a stroke patient, you ask bim to stick out his tongue. At first he is unable to do this, but a few moments later he performs this movement spontaneously. ‘This defect is known as A) apraxia B) agnosia C) expressive (Broca’s) aphasia D) astereognosis En évaluant un patient sttaint d’an accident vesiculaire ‘cérébral, vous lui demandez de tier Ja langue. Au début, il est incapable de le faire, mais quelques instants plus tar, il ceffectue ce mouvernent spontanément. Ce défaut est connu sous le nom A) apraxie B) agnosie ©) apbasie «expression (Broca) D) astéréomnosie ial ay Ge ile Ras Goan ple (cUial ae a oS ols ayo phe YL gh ara ul i PASS a al 8 as A ac -B Lies (Sapien peaive D 23 ‘A 45-year-old school teacher presents with a S-month history of hoarseness that isnot improving ‘The most appropriate management at this time would be A) voiee therapy B) azithromycin )a tial of inhaled conticasteroids D) a tal of a proton pump inhibitor ) laryngoscopy Un professeur d'école de 45 ans se présente aver une histoire de 3 mois dienrouement qui ne s'améliore pas. Le traitement le plus appropri en ce moment serait A) Un traitement de la voix B) fazithromycine ©) un essai de costcostéroides inbalés D) un essai d'un inhibiteur de la pompe & protons E) laryngoscopie YBa uA) gis Ce £6 eae jae ga gh Aba aia Lae SS ual Gf oe Capaldi “A eels) -B i hy pa 6s ole as -C iy Bh Rn Ae bel as -D Sypia jhe cla! -E 24 Which one of the following would be most appropriate in the management a 15-month-old male with suspected uncomplicated community acquired penurnonia seen in an outpaticat clinic with less than 24 hour iliness. A) Hospital admission and sepsis workup B) Inpatient monitoring, with no antibiotics at this time C) Hospitalization and intravenous ceftriaxone D) Close outpatient follow-up, with no antibiotics at this time E) Oral high-dose amoxicillin (90 mg/kg/day), with close ‘outpatient follow-up Lequel des choix suivants serait le plus appropri dans fa gestion d'un enfant de 15 mois avec suspicion dune enumonie acquise dans la commmunaute et qui est vu dans lune clinique externe avee moins de 24 heures de maladie. ‘A) Admission a Vhépital avec bilan de septivémie B) Admission a lhopital et observation sans antibiotiques en ce moment ) Hospitalisation et de la ceftriaxone par voie intraveineuse D) Suivi ambulatoire rapproché, sans antibiotiques a ce moment, E) Amoxicilline (90 mg / kg / jour), par Voie orale & dose élevée, avec un suivi ambulatoire rapproché ial aa pa VLD SEV a GID Ge gl We RnB Cle Lane AS ing Ligh V8 ene YE gel ans Sap JS Sal jb aS Cane go i Teal ge tele p28 gaged Lyall gies A ge as Ab py gal yt Jas) -B SNe tie Gay Sl je elles ell -C poe dan Jail lp Myla! Aladl -D BSN adap Cine elbl mS on tapad ihe He yn dhe! -E hall gb all dal aa (@/S/be) pore) 25 26 Tr children fixed splitting ofthe second Heart sound is characteristic of A. Patent ductus arteriosus B. Small ventricular septal defect C. Ebstein's anomaly D. Atrial septal defect E, None of the above. Le dédoublement du 2°" bruit cardiaque chez les enfents est trés en faveur de A. Persistance du canal artériel B, Petite communication interventriculaire C. Anomlaie d’Ebstein D. Communication interauriculaire. E, Toutes les propositions sont fausses PR Se AB Spal I JIS g Chinaly ce sa Apa) - A ital lal ys ae ue -B cays -C ei Jalal gs coe -D. Bane oe ela RMS -E A 48-year-old woman has abnormal vaginal bleeding. Her periods arc lasting 3-5 days longer than usual, bleeding is heavier, and she has experienced some intermenstrual bleeding, Her physical exam, hemoglobin are normal and pregnancy test negative. Which one of the folfowing is the most important next step in management? A) Laboratory tests to rule out thyroid dysfunction B) An endometrial biopsy C) Oral contraceptives, 4 times a day for 5-7 days 1D} Cyclic combination therapy with conjugated estrogens (@remarin) and medroxy-progesterone (Provera) each month, E) Administration of a gonadotropin-releasing hormone analog such as leuprolide acetate Une femme de 48 ans a des saignements vaginaux anormatx Ses régles sont dune durée 3-5 jours plus que d'habitude, le saignement est plus ebondant, et elle @ connu quelques saignements intermenstruels. Son examen physique et Mhémoglobine sont normaux et le test de grossesse est neégatif, Lequel des énoncés suivants est 'étape la plus importante ans la conduite & tenis? ‘A) Les tests de laboratoire pour exclure un dysfonctionnement thyroidien B) Une biopsie de l'endométze ©) Les contraceptifs oraux, 4 fois par jour pendant 5-7 jours 1D) thérapie de combinaison eyclique avec des cestrogénes ‘conjugués (Premarin) et medroxy-progestérone (Provera) chaque mois, B) ladministration d'un analogue dhormone libérant la gonadotropine tel que Vacézate de leuprolide atte oe line GID) cp lid le Ea pe tL SR gash Ge Ah arg Spb eG pence eant outa 9) ay BUYS Sia VE Be gh gl ash hy gail Gnas galt ol gh Sal SY Al soil Aiday gi I aay ke Gasad A rar Byte ba IVE Bd pslly Oe Fy on lye iy 34 IS bans hfe Che a Sin gS p84 Ha cpg Sel Fal ll ye ee AL ee A sty ah Se cel 27 You have recently removed @ nevus fom a patient. The pathologist reports the lesion as a dysplastic nevus (an atypical mote). What do you suggest to the patient? ‘A. Re-excision with a 2 to 3 mm margin to ensure complete excision Nothing Observation Reassurance Re-excision with a S-cm margin muoD Vous avez révemment excisé un nacvus d'un patient. Le pathologiste signale Ia lesion comme un naevus dysplasique (un naevus atypique). Que proposez-vous pour le patient? A. Re-excision avec une marge de 2 4 3 nim pour assurer une exérése complete B Rien ©. Observation D. Réassurer E. Re-excision avec une marge de $ om rel aa Ga ee LA SI 1a (gy ga) Gath A any 2 BTC uaa ge ci St pT gh ad Jt het =A easy ph ed B aly sips pe? pb tha Dents 28 ‘Uncomplicated, type B (distal) aomtie dissections are usually treated with which of the following? Anticoagulants Blood pressure control Surgery Intra-aortic balloon pump None of the above mIOw> La dissection aortique non compliquée de type B (dstale), est généraloment traitée avec laquelle des suivantes? A. Anticoagulants B. Le controle de pression artérielle Chirurgie D. pompe & ballonnet intra-aortique E, Aucune de ce qui précide eA Dayne Jane at ely JS -E 2 ‘A 41-year-old man fas been drinking up to 20 liters per day ‘of water for the past week. His lab shows sodium 162 mmoV/L. potassium 4.1 mmoV/L, chloride 121 mmol/L, bicarbonate 27 mmol/L, glucose 75 me/AL, creatinine 1.0 mg/dL, and serum osmolality 343 mOsm/kg, A deficiency of ‘which of the following hormones is most likely present in this man? Antidiuretic hormone Oxytecin Insulin Growth hormone Prolactin moOD> Un homme de 41 ans buvait jusqu’ 20 litres d'eau par jour durant [a semaine passée. Son bilan de laboratoite montre sodium 162 mmol /L, potassiam 4, mmol /L, le chlonure de 121 mmol/L, le bicarbonate de 27 mmol /L, glucose 75 mg/dL, la créatinine de 1,0 mg/dl. osmolalité du serum 343 mOsm / kg. La deficience de quelle hormone est probablement présente chez ce patient ‘A, hormone antidiurétique B. Ocytocine C. Insuline D. hormone de croissance E. prolactine Gey ahs gle a ttle Oee day Basal yaad Ce gag DIS EY pang lle YTV pp pal YY hig Salle Ala 10) a3 i ae 9 ty J aaa VO 5 Salle VU ele Gt Slee PET onal yan ae Via ae 5 EM gt GI alg ye tas oth Nay ead opel “A, Ome! -B cleat -C silane -D oy EB 30 ‘A 26 y.o-female presents with sudden onset of mental confusion and subsequently seizure. Lumbar pucture shows 1 RBC and 20 lyrophocytes, with normal glucose and protein. MR imaging of ber brain reveals swelling of the right temporal lobe with hemorthagic areas, The most likely etiologic diagnosis is: Mycobacterium tuberculosis Herpes simplex virus Influenza virus Hemophilus influenza Brucelia PoORE Une femme de 26 ans se présente avec une apparition brutale d'une confusion mentale et par la suite une convulsion. La ponction lombaire montre | globule rouge et 20 lymphocytes, avec le glucose et la protéine normale. IRM de son cerveau révéle un edeme du lobe temporal droit avec des zones hémorragiques. Le diagnostic etiologique le plus probable est: A. Mycobacterium tuberculosis B. Herpes simplex C. virus de la grippe D. Haemophilus influenzae E, Brucella conti viylln ghgay Siyuae Gls We A SHS il ih A) GAS ey it lad Sa grad Sy p29 SSE Ai V6 gel yas ea ail bp Ela pain 5 I eae FYI cel path ob A clan en al abs derbi A dual tals BB ‘Bk C plc hese! E 31 | Epidural anesthesia in a pregnant woman may result in all of the following conditions EXCEPT ‘A= Hypotension B Decreased placental perfusion C- Decreased venous return D- Increased venous pooling | E- Increased cardiac output Lianesthésie péridurale chez une fesnme enceinte peut entrainer toutes les conditions suivantes SAUF: ‘A- Hypotension B- Diminution de la perfusion placentaire C- Diminution du retour veineux D- Augmentation de la stase veuineuse E- Augmentation du débit cardiaque SMG Ke Da aaa 5 Sl ie LY CYL dia Ualis) -A etal el Ila Gat 62s) oll galls oa anal alll gS gi 9 -E. 32 | An a-fetoprotein of 0,44 times normal would predict the possibility of which of the following conditions? ‘A; Omphalocele B- Trisomy 21 C- Multiple pregnancies D- Trisomy 30 E- Tumer’s syndrome Une a-fetoprotein de 0,44 fois la normale prévoit le possibilité de laquelle des conditions suivantes? ‘A. Un Ompbalocéle B- La tnsomie 21 Ce grossesses multiples D. trisomie 30 E. syndrome de Tuner Ge ail oo Blin EE os) pha Gl SE GMA ye gl yom ial tg} ue 1 gall aaa J Ts pineal 5 soba -E 33 AT month old infant was brought tothe ER at 4:00am for respiratory distress and cough. Physical exam revealed expiratory stridor with sub costal retraction. His temperature ‘was 38.5C. The patient most probably has: ‘A. Broncho-pneumonia, B. Asthmatic atack C. Laryngotracheo bronchitis. D. Foreign body aspiration E. None of the above Un noumrisson de 7 mois a été amené aux urgences 4 4h00 du matin pour une détresse respiratoire et une toux. L'examen physique révéle un stridor expiratoire et rétraction sous- costale. Sa température est de 38,5 C. Le diagnostic le plus probable est: A. La broncho-pneumonie B. Une crise d’asthme C. Une Laryngo-trachéo-bronchite. D. Aspiration d'un comps étranger. E. Aucun des précédents Fetal Se esha pal) puns ygetl Vo ae ile Oi ly Api Mtn ya ing gy Cabin dal SS BUS a8) ne yg ell oan Niglan s SSYI ALayh GL go PAS Smell umn ees I “A ee B Ae gat hah het He penal) -D Fae ye sel dpi US -E 34 A [2-year-old boy presen with left hip pain and @ limp. He says the pain started gradually after sports game without any injury. He is afebrile and has limited internal rotation of the lef hip. What is the most likely cause of the hip pain” A) Septic arthritis B) Juvenile rheumatoid arthritis C) Transient synovitis D) Slipped capital femoral epiphysis E) Legg-Calvé-Perthes disease Un gargon de 12 ans se présente avec des douleurs de la hhanche gauche et une boiterie. I! dit que !a douleur a commencé progressivement aprés un jeu de sport sans aucun traumatisme. Il n'a pas de fiévre eta une linntation de la rotation inteme de la hanche gauche ‘Quelle est le cause la plus probable de la douleur la hhanche? A) Llartrite septique B) Laarthrite rhumatoide juvénile C) une synovite transitoire D) Une épiphysiolyse du condyle fémorale E) Maladie de Legg-Calvé-Perthes EA 9 rN a gh le Sou Ue Ve at gt Lely 9 Aad ey Gan oa JY Gy a ad ASIA oLaplly5 Joe Ge Unt Ae Ga VG 5 BS Gul apie a aL ays itl Santini A hth gts dene heal le BL el ‘el Sak p38 Samy ieb -E 35 ‘A serious complication of the treatment of bypematremic debydration is: A. Cerebral thrombosis B. Cerebral edema C. Hyperchloremia D. Hypoglycemia E. None of the above Une complication grave du traitement de la déshydratation hypematrémique est la suivante: A. Thrombose cérébrale B. Liedéme cérébral C. Hyperchlorémie D-Hypoglycémie E. Aucune de ce qui précéde poRpal lip Gina glia a yh MEL al ele BS eis, i oas ot faye yf Del diyg i JS OE cos 36 ‘A 52-year-old patient is concemed about a biopsy result from 4 recent screening colonoscopy. Which one of the foilowing types of colon polyp is most likely to become malignant? A) Hyperplastic polyp B) Hamasiomatous polyp ©) Tubular adenoma 1D) Villous adenoma E) Tubulovillous adenoma ‘Un patient de 52 ans est préoccupé parle résultat d'une biopsie lors dune coloscopie de dépistage recente. Lequel es types suivants de polype colique est le plus susceptible se devenir malin? A) Polype hypemplasique B) Polype hamartomateux ©) Adenoma tubulaire D) Adenome villeux ) Adenoma tubulovilleux WSR Rk ELE Te ae JS ge SAIL Ml oa gt OS he 1s geal yada the augmentation des reflexes tendineux est due & une atteinte ée A.La moelle épiniére B. Le nerf périphérique C.Le cervelet D. Les récepteurs tendineux a l'étirement E.Les muscles ee AYE A LSE al CLASUT Bis! Stl La “A cael eat B ead -C gl teh -D Dua -E 40 Which one of the following is most typical of polymyalgie rheumatica? ‘A) Headache and neck pain B) A normal erythrocyte sedimentation rate (ESR) C)A dramatic response to corticosteroids within 72 hours D) A lack of systemic sympioms and signs E) Weight gain and hyperphagia Leque! des énoncés suivants est le plus typigue de la pseudopolyarthrite rhizomélique? 'A) Maux de téte et douleurs au cou B) Un taux normal de sédimentation (VS) ‘C) Une réponse spectaculaire aux corticoides dans les 72 | heures D) Une absence de symptomes et de signes systémiques E) Le gain de poids et boulimie Shad A Mle SSW p Gdn gh Stes EM gi db glne -A th pel oly Si aeons -B VY DS cig a Ss STS tate -C jel Reali ey 2M Best -D sAa a3 bth -E coi? a Which one of the following is the greatest risk factor for abdominal aortic aneurysm (ANA)? A) Cigarette smoking B) Diabetes mellitus © Vasculitis D) Female gender Lequel des énoncés suivants est le plus grand facteur de risque de Tanévrisme de l'aorte ebdorinale (AAA)? A) La cigarette B) Le diabéte sueré C) Vasculite D) Le sexe férinin PRD pal AY aS gal Ll a iG oo | gal gos A Sales -B we yell -C p89! oda -D mR A 65-year-old alcoholic male is admitted with bacterial pneumonia. The sputum culture grew out gram-positive alpha hemolytic diplococci, catalase negative, and bile esculin positive. The causative organism of this patient’ pneumonia is A. Staphylococcus aureus B. Streptococcus pyogenes C. Enterococcus faecalis D, Staphylococcus epidermidis E. Streptococcus pneumonia Un Bomme alcoolique de 65 ans est admis avee une pneumonie bactérienne, La culture des crachats pousse en diplocoques a Gram positif alpbe-hémolytique, catalase négatif ct positf la bile esculine. L'orgavisme responsable de la preamonie chez ce patient est A. Staphylococcus aureus B. Streptococcus pyogenes C. Enterococcus faecalis D. Staphylococcus epidermidis E, Streplococcus pneumoniae erbay hg pine gal pha Le 18 ne Say Sic sans Cll E13 Oe Oat Rade BOO sagt SAS oh Agha gpl da da p30 IU pea gyal Ob dnl gal eal GA eda 3) tik ae ya i gill cy Sal A Seals St -B oll yeall cS) -C pall Ap ll $0) -D Wp ill Sa) ‘Which ofthe following findings of chest ray i most consistent with the diagnosis of chronie obstructive pulmonary disease (COPD)? ‘A. Hyperinflation, bronchial thickening B. Perihilar fluffy infiltrates C. Pleural effusion, Kerley B lines D. Diffuse nodular opacities E, Ground glass shadowing A la radiographie pulmonaire, lequel des résultats suivants, est le plus compatible avec le diagnostic de la maladie pulmenaire obstructive chronique (BPCO)? A. Hyperinflation, épaississement bronchique B. Infiltration flocconneuses périhilaies €. Epanciement pleural, lignes de Kerley B D, Opacités nodulaires Diffuses E, Verre dépoli A PE gepall AID sl On gh Tope gs slay od path a Kynal Slee gil bs =A oft nae thi CHALE -B ea lS bash eyes chet -C 5 pia kde take -D Ayjledplaj ah -E 44 Mastoiditis usually results as a complication of which of the following infections? A. Parotitis B. Dental abcess, C. Acute otitis media D. All of the above La mastoidite résulte habituellement comme une complication de: A. La parotidite B. Labeés dentaire C. L’otite moyenne aigué D. Toutes les propositions sont justes CHAS Ge GY ie aS ie CARD A as ASD A A, eels -B da gesost tell -C Aameedel oils -D 45 The sigmoidoscopy report of @ 70-year-old man shows melanosis coli. He has a long history of constipation, hypertension, and diet-controlled type 2 diabetes rvellitus You would now A) perform a metastatic workup B) review his medications C) prescribe oral corticosteroids D) prescribe antibiotics and a proton pump inhibitor E) check his stool for Closiridium difficile Le rapport sigmoidoscopique d'un horame de 70 ans montre tune mélanose colique. Ila une longue histoire de constipation, dhypertension, et un diabeéte de type 2. Vous allez le controler parle régisoe alimentaire maintenent. ‘A) effectuer un bilan métastatique B) passer en revue ses medicaments © preserire des corticaides par voie orale D) prescrire des antibiotiques et un inhibiteur de la pompe & protons E) recherche de Clostridium difficile dans les selles te Vea pe aS gin al Belt otha yet eal golly! Bl Vg Say tal 8 J a Ci ull Asie dans Mice gh “A Saplings -B a ys Sip Steg -C sia aa ay ype Ciba ay -D BoD gi teal Ala oe yl -E c-19 4% ‘A patient complains that he develops shortness of breath and ‘wheezing wien he goes jogging or engages in other heavy ‘exercise, but has no breathing difficulty otherwise. The most likely explanation for this history is which one of the following? A. He is allergic to something in the environment ‘where he jogs He is allergic to some component of his athletic. ‘equipment He has exercise-induced asthma The psychological stress associated with ‘exercise triggers latent asthma Dyspnea and wheezing are normal responses to heavy exertion m 90 B Un patient se plaint qu'il développe un essoufflement et des sibilants respiratoires quand il va faire du jogging ou se livre 4 un exervice intense, mais n'a aucune difliculté & respirer autrement. Liexplication la plus probable pour cette histoire est A. ihestallergique & quelque chose dans environnement oi il fait du jogging, B, Il estallergique & certins composants de son équipement sportif C.tlaYasthme deffort D, Le stress psychologique associé a l'exercice déclent Vasthme latent E. Dyspnée et sibilants sont des réactions normales @ un effort intense Ay ap sie Seiya ppnad igs Ge Se sare Ayyaee coe Sta Y Sy titans pls BAS Ll eal AS! on yh SMA SA tS ual Yi fea Nd a i 89 By SIE GE eyo es A Beaty Mane LS gia yen AB Guat ane rata oC a sir caslall eo all padi SIG DD Pres Sale Uae ga SOM oath Sade! seh a in a7 ‘A 60-year-old otherwise healthy man presents with, symptoms of reflux disease and undergoes an endoscopic evaluation, The biopsies in the distal esophagus arc consistent with high-grade dysplasia associated with Barrett esophagus. What is the appropriate treatment for this patient? ‘A. Proton pump inhibitor therapy B. Esophagectomy C. Anti-refiux surgery D. Observation with annual endoscopic gastroduodenoscopy Un homme age de 60 ans en bonne Sante se presente avec des symptomes de reflux et subi¢une evaluation endoscopique. Les biopsies au niveau de I'cesophage distale ‘montrent une dysplasic de havt grade associce a ua eesophage de Barrett. Quel est le traitement approprie chez ce patient ‘A. Traitement aux inhibiteurs de la pompe a proton B. Oesophagectomie C. Chirurgie anti reflux D. Surveillance avec une gastro duodenoscopie annuclle al yel ce Sty hae Mane Gey Like Vs 6 pe Jay, agit pill yo Last sal Sy play ab Ea Ge Re Sa gall gpl le FS Tag) ah lg La eg il gin Fal sig pS Akan tla Glee A, gpl deal BB epee oC oie gb phi eLalgedsal DD 8 Oral candidiasis can be found in A. Infants, B, Children with AIDS C. Children receiving antibiotics D, Children with nutritional deficiencies E. All of the above La candidose buccale peut étre trouvée chez: A.Les nourrissons B. Les enfants atteints du sida C, Les enfants recevant des antibiotiques D. Les enfants présentent des carences nutritionnelles E, Toutes ces reponses ae pai aga ol any goSe east A JePL os healt Jab! -B Hayy Oban shh el DabN -C Sey shane ibd =D dagen Mel Ayal SE 33 Diabetes insipidus may be due to all of the following except A. Pituitary adenoma B. Renal epithelial ADH reception defect C. Hypokalemia D. Hypercalcemia E. Adrenal deficiency Le diabéte insipide peut eve due a ensemble de ce qui suit, sav? A, Adénome hypophysaire BB. Défaut de réception ADH au niveau de lepithelium renal C.Linypokaliémie. D. Libypercalcémie E. insuffisance surrénalienne OS ce Sg A Sons Re wig es “A Bis Jalil BADH Ue ie -B Ph aig ott pbs h D BGI oped -E 30 ‘A 3-yi-old girl presents with microcephaly aad mental retardation, Her mother had a f-like illness during the second month of pregnancy. At birth, the baby bad petechiae and hepatosplenomegely, which have resolved, The most likely diagnosis is: ‘A, Congenital HIV infection B. Congenital rubella C. Congenital parvovirus infection D, Isoimmune neonatal thrombocytopenia E, Subacute sclerosing panencephalitis Une fillette de 3 ans a présente une microcépbatie et un retard mental. Sa mére a eu un syndrome pseudo-grippal pendant le deuxieme mois de grossesse, A la naissance, le bebe avait des pétéchies et une hépatosplénomégalie, qui ont résolu. Le diagnostic le plus probable est Ia suivante A. Linfeetion congénitale SIDA. B. rubéole congtnitale. Cnfection a parvovirus congénitale D. thromboeytopénie néonatale iso-immune E, La panencéphalite selérosante subaigué gle AIS y ly pine gaa Ci ge Us jae Mie I AML A ip lly a boyy Mall pea NM Se Dal Ge IS) pepbal of Aaa cay ay Dale pay Abas iS HIV Oa =A Wada -B AISI hel Ayal gis) -C Ly Aapal Splines tb -D ted cans al SLA ELM Aga 31 Prophylactic antibiolies after a bite is especially recommended: AcIn cat bites B-In case of human bite C-If the wound is extensive o facial Din case of joint damage or bose E-All answers are correct, La prophylexie par antibiotiques aprés morsure est surtout recommandée A> En cas de morsure de chat B- En as de morsure humaine C- Sila plaic est extensive ou faciale D- En cas d'atteinte articulaire ou osseuse E- Toutes les réponses sont correctes [Aaa ny Gilg gg Se the kp Lit ine as Prise de G- Bloguants B. Sclérodermaie C- Prise de Nifedipine 1D. Maladie de Buerger ou thromboangtite oblitérante E+ Lupus érythémateux dissérin€ The Ugh ds ga o87 gu alla! Gill hala Eada =A, abl kes -B Caps Aaeal -C Seng HE gle se I tyes -D Awad -E @ ‘Arterial blood gases froma 75 year-old man, emphysematous for 20 years, show: pH = 7.35, p02 = 50 mmHg, pCO2 = 60 mra Hg; Bicarbonates = 33 mmol/|. This A. Metabolic acidosis B- Respiratory acidosis C-Respiratory and metabolic acidosis D- Mixed alkalosis E- Metabolic alkslosis, Vous faites des gaz du sang arttriel a un homme de 75 ans, emphysémateux depuis 20 ans. Ils montrent : pH= 7,35; pO2=S0mmilg; pCO2=60 mm He; Bicarbonates= 33 mmotes/ sagt dune: ‘A- Acidose raétabotique B- Acidose respiratoire C- Acidose respiratoire ot métabolique D- Alcalose mixte E- Alcalose métabolique Ba gal LIEV oper ah gh pal pS Gl 135 pa + = POD TS = PH: Saab ste Ts SUSap gle YY = i Sie edt pe Vs = POD flit Gunns A wt gees B erty i oan -C tata -D toi G8 B 64 Which of the following hemolytic diseases may be precipitated by aspirin intake? A. Inherited sickle cell anemia B, G6PD deficiency anemia C. Aplastic anemia D. Megaloblastic anemia E. March anemia Laquelle des maladies hémolytiques suivantes peut-clle étre déclenchée parla prise daspirine? A, Lianémie héréditaire (Sickle cell) B, Déficit en G6PD CC. Lanémie aplasique D, Lianémic mégaloblastique E, Anémie la Marche Tew Ons) a RAD Galo gh Semel pascal Barge BOA GOPD bye 8 -B wie HC hy M Sp -D vel BE ‘65 A patient with chronic Kidney disease presents with Chronic ga 8 qu SH yay Cm a go Gm gL normocytic anemia with @ hemoglobin level of 7.8 g/dL. cash funn WHE LE NEVA cs ya (2 Rann (E.G Ga ie 1 is predicted if you rai The best outcome is prested if you rae te beroglobin asa ey ena gl A) 8-10 g/dL se B) 10-12 g/dL. Dee AA ©) 12-14 gid wre D)>14 g/dl deg, B Un patient ateint de maladie rénale chronique présente une anémie normocytaire chronique avec un taux dlaémoglobine de 7,8 g/ GL. Le meilleur résultat est prédit si vous augmentez le taux dik’moglobine a A) 8-10 g/dl B) 10-12 g/dL ©) 1214 g/dl D> 14 g/dl 66 | In streptococcal toxic shock syndrome, all the followings are true except one: ‘A. Renal impairment B. Respiratory distress syndrome C. Soft tissue necrosis D. Pancarditis E. Coagulopathies Dans le syndrome de choc toxique streptococcique, toutes les propositions qui suivent peuvent s"observer, sauf: A. Atteinte rénale B, Syndrome de détresse respiratoize CC. Nécrose des tissus mous D. Pancardite E, Coagulopathies La ee gh US Signy pa Ran De le AS clits oA Cot on De -B, te Melly As -C ALN ll gst -D Ss ME -E Risk factors for pancreatic cancer inelude all but the following: ‘A. Hereditary pancreatitis B. History of pancreatic cancer in 2 first degree relatives over age 60 Smoking Alcohol BRCA2 mutation moo Les facteurs de risque pour le cancer du pancréas comprennent tous les éléments suivants SAUF. A. panertatite héréditare B. Histoire du cancer du pancréas chez 2 parents de premier degre agés de plus de 60 ans C. Tabagisme D. alcool E. Mutation BRCA2 Leh SA Sill a pa 3g al ll gens be Pa Se heal -A Se DM Ea AS hm eB. he Vs Bok pt eel HM) Ge GL osu -C das -D BRCA2+ib -E, e ‘A 17-year-old feraale patient came to the ER with symproms and signs of bacterial gastroenteritis. Which of the following is the causative organism that raimics symptoms of acute appendicitis? A. Yersinia enterocolitica B. Yersinia pseudotuberculosis C. Yersinia pestis D. Francisetta tularensis E, Pasteurella multocida ‘Une patiente de 17 ans est venue a l'urgence avec des symptdmes de gastro-entérite bactérienne. Lequel des germes suivants est le micro-orgenisme qui simule les symptimes de Tappendicite siguée? A Yersinia enterocolitica B. Yersinia pseudotuberculosis C. Yersinia pestis D. Francisella tularensis E. Pasteurelia multocida hse ad ll eae le VY pe ih Ft Ania haa ap lam CH fly oil gel Oe gh hell ae Ak lh Gaal all 9h A saat ls Galas oleae ined “A 4981 Shall -B Rell eed -C Aa Ail -D ay sald -E o ‘A 36.0. heavy smoker male presents with weight loss and dyspnea on exertion. He reported exposure to asbestos at work, with appropriate respiratory precautions. CXR reveals 4 hilar cavitary mass 4 centimeters in diameter. The most likely neoplasm this man has is: Hamertoma Bronchoalveolar carcinoma Mesothelioma Squamous cell carcinoma ‘Small cell carcinoma moow> Un homme de $6 ans, gros fumeur présente une perte de poids et de la dyspnée a l'effort. Il a signalé exposition & Vamiante au travail, avec les précautions appropriées respiratoites, La radiographie pulmonaire révele une masse hilaire cavitaire de 4 centimetres de diamétre, La tumeutr la plus probable de cet homme est ta suivante: A. Hamartome B. Carcinome broncho-alveolaire C. Le mésothétiome D, Carcinome épidermoide E, Carcinome a petites cellules OF BE pam ohh le OV ope dey a geal ag se Gab Sate 5 gui CHa Sal ga ae LS Cady poet ship ARS 1M yell papal ll Ail A aS se Gan 5 ASU a tl oh A muows 70 Which of the following is true about Philadelphia chromosome and chronie myelocytic leukemia (CML): ‘A. Philadelphia chromosome refers to an altered chromosome 20 B. Philadciphia chromosome is seen in the bone marrow of over 90% of persons with CML C. InCML, most patients have blasts more than 30% in peripheral blood on presentation D. in CML, there is translocation of latency genes between chromosomes E, None of the above Laguelle des propositions suivantes est vraie au sujet du chromosome de Philadelphie et Ia leuctmie myéloide chronique (LMC): ‘A, Chromosome Philadelphie se réfee & un chromosome 20 modifié B. Chromosome Philadelphie est va dans la moelle osseuse de plus de 90% des personnes atteintes de LMC C. En cas de LMC, la plupart des patients ont des blastes & plus de 30% dans le sang péripherique a la présentation . En cas de LMC, il ya translocation des génes de latence entre les chromosomes E, Aucune de ce qui precede ales GAD ginal ale came gl Om Foal Sil el Puce ite pL YGtne A Weal SY gala gL JU yosen ty B iia coed pal2Y G4 968 Ge orl gal pl ans nal gil pl pking! la gi C WE ce BSC I te OS eel enllayt Da jal og i pl slain Ma ys DD etal oe ESI at ge as fone ye Dela E 7 ‘A 36-year-old male with history of caleiums oxalate stones diagnosed with primary hypertension. Which of the following class of antinypertensives might best be avoided due to his history of nephrolithiasis? ‘A. Angiotensin-converting enzyme (ACE) inhibitors Bata blockers Calcium channel blockers Loop diuretics Tiiazide diuretics moaw Un homme de 56 ans ayant des antécédents de caleuls doxalate de calcium est diagnostique avec hypertension primaire. Laquelle parm les classes suivantes Une jeune fille de 14 ans développe une diarrhée qui s’aggrave progressivement associée 4 des douleurs abdominales, une perte de poids et de la fiévre. Les selles, conticanent du mucus, du sang, et des globules blancs, mais ‘pas de parasites ni des bactéries pathogénes. Le avement baryté montre des uleéres séparés par des muqueuses saines ‘Quelle est la pus grande préoccupation pour le pronostic & Jong terme de vette maladie? A, Ocelusion intestinate et fistules B. Carcinome aprés 10 ans de maladie C. Arthrite déformante D. La spondylarthrite ankylosente E, Le mnégacolon toxique TSE) Gee ops Deal Wh can le VE Laue $509 BL Ey 9 Ds ge gly lly ogee 29.9 a ean 92 MS yop de le pa Ha Gil babe ah pte ee dale 9 pL Rabe Glo pee ye Mati le fi Viel so al SSW gh JY Ga ay Suey cela ma cal OA Rail ie oie te te B apie Jee el C diel sites DD odes B The most definitive tet for identifying intracranial aneurysms is ‘A. MR scanning B. CT scanning C. Single photon emission computed tomography (SPECT) D. Positron emission tomography (PET) E. Angiography Le test definitif pour identifier les anévrismes irtracraniens est A. MR sean BCT scan C. Single photon emission computed tomography (SPECT) D. Tomographie par emission de positons (TEP) E. angiographic Tate pal ple AA as JW Gaal * gettblibe gas A ue git owe -B sigh Joa ney gate eget -C os Fajzll jal gabe gpa -D. BM peat EB 74 A 69-year-old female with recurrent palpitations since menopause, is started on quinidine 200 mg every six hours. One month later, she sustains two episodes of near-syncope, with torsade de pointes documented. The next step is ‘A. Obtain a quinidine level B._ Send the patient to the emergency room for a head CT scan C. Send the patient to the emergency room for ‘magnesium sulféte infusion D. Send the patient to the emergency room for overdrive suppression with atrial or ventricular pacing E, Stop the quinidine ‘Une femme de 69 ans avec palpitations a répétitions depuis sa ménopause, a débuté un traitement par quinidine 200 mg toutes les six heures. Un mois plus tard, elle souftie de deux Episodes de presque-syncope, avec des torsades de pointes documentées. La prochaine étape est la suivante: A. Obtenir un niveau de la quinidine B. Envoyer la patiente a la salle durgence pour un scanner de la tee C. Envoyer la patiente A la salle durgence pour une perfusion de sulfate de magnésium D. Envoyer Ja patiente ia salle durgence pour réduction de Varythmie par stimulation auriculaire ou ventriculaire E. Arte la quinidine (Us) Sie 9 Sin iS Ge ghd le pe Al hele SS gains ga Yes deca «Cad asian ta all S Gs ee ee A AIS SSG! Salsa CAT sien gle Spall A Mad as slag Maw pda) B gape git coe Say as SRNL May lh Jeesd oC peal Ob Sd dal oa es sie iat D 3h AM An IB Gn Sea oe hall eal 75 While riding bis bicycle, a 9-year-old boy loses control and falls. During the process, his abdomen strikes the handlebar. iis parents bring him to the emergency department because hehas vague midabdominal pain and some bruising of the anterior abdorninal wall. His vital signs are stable, and be has no otter visible injuries. Which of the following is the most likely diagnosis? ‘Ruptured spleen Ruptured liver Ruptured pancreas Hematoma in the rectus muscle Ruptured duodemum moom> Un jeune de 9 ans, tombe apres qu'il a perdu le contrale de sa bicyclette. Durant la chute, son abdomen cogne le guidon. Ses parents l'amenent au departement d’urgence a cause d'une douleur vague au milieu de I’abdomen. Ses signes vitaux sont stables, et il n'a pas d’autres lesions visibles. Quel est le diagnostic le plus probable rupture splenique rupture hepatique rupture pancreatique hematome du muscle grand droit rupture duodenale mpOB> GS) A La ghay SE Cg Va ae 1a ale pS Je By ae Aa yl Aino tel ae lls OW 5 ll pall lly tal pA lal jam gh chess ines Sha dey gf oe gh ag ph Sail al aly 6 iy pol Lt Naan eS ask Bal Jab abs -A S55 -B bse dss -C pal) tae hegd eas -D gre pas -E 76 ‘Which of the following is NOT a cause oF hypertriglyceridemia? ‘A. Acromezaly B. Alcobolism C. Diabetes mellitus D. Estrogen therapy E. Hypothyroidism Lequel des énoncés suivants n'est pas une cause de Vhypertriglycéridémie? A. Lracromégalie B. Alcoolisme C. Le diabéte D. Liestrogénothérapie E. Lhypothyroidie Fpl eal pS A ee ad ok I Ue gs al “A Usa ge oe! -B p&alles -C Cos ANG Ape -D Sb Gat -E c-2 77 ‘A 57-year-old septic male patient is suspected to have & diverticulitis with probable perforation. Which of the following studies would best confirm the diagnosis. Barium enema Colonoscopy CT scan of the abdomen Flexible sigmoidoscopy Sonogram nome Un homme de 57 ans, septique, est soupgonné ¢'avoir une diverticulite avec perforation probable. Quel est J'examen qui confirmerait le mieux le diagnostic A, Lavement baryte B. La colonoscopie C. Le scanner de 'sbdomen D. La sigmoidoscopie flexible E. Echographie CHa ol a yy Ge Gas le OV ewe a OF SRE Hell eg Uae A gs Fel Jt as a5 paesbitie A oss ke -B hi gare ith ae -C 03. gine as -D pall 39 Cha ye hs -E 78 Which one of the following statements about plantar fasciitis is tue A) Its a coramon cause of metatarsal tenderness B) Pain is located on medial plantar aspect of heel ©) Pain gradually worsens when climbing stairs 1D) Xrays are necessary for correct diagnosis, £) Steroid injection is the preferred treatement Lequel des énoneés suivants au sujet de la fascite plantaire est vrai ‘A) Ii s‘agit d'une cause commune de fa douleur métastarsienne B) La douleur est située du coté plentaire et interme du talon C) La douleur staggrave progressivement lors de la montée des escaliers D) Les radiographies sont nécessaires pour un diagnostic comrect E) injection de stéroides est le traitement de choix Sess A AY Seay A al Gh Fine pel SU CA ala A ee basil tall le NM gas B ‘ial De gle peat ae GeiMer C ON ortaype telat pal as D. all yas Anal deal ga Sty id ci E 79 ‘The Glasgow coma scale measures consciousness by rating parameters that include all the following EXCEPT Orientation Breathing Bye opening Limb posturing Speech noOw> Liéchelle de coma de Glasgow mesure la conscience par des paramétres suivants SAUF ‘A. Orientation B. La respiration C. Ouverture des yeux D. Position des membres E. Paroles A al le ge I a A Rane LO! Ha gba dS Gus oud A, dial -B wate -C SibY pay D ge 80 ‘A 52-year-old woman with microoytic anemia and a long standing history of constipation has occasional hemorrhoidal bleeding, but no melena, and guaiac positive stool on rectal exam without any masses. What is the most useful next diagnostic test for this patient? CT seen Plain abdominal X-rays Colonoscopy Exploratory laparotomy Upper endoscopy mMoOm> Une femme de 52 ans présentant une anémie vierocytaire et une fongue histoire de constipation, a occasionnellement des sbignements hémorroidsires sans méléna, avec gaiac posit. Pas de masses au toucher rectal. Quel est test de diagnostic suivant le plus utile pour cette patiente? A.CT sean B. Radiogrepbie abdominale simple C.La eolonoscopie D. Une laparotomie exploratrice E, Endoscopie digestive haute SUSI gee Bo gle oT Ae Hl ERAN eel Oekabna red seey SMG yY hel esd mamgene il A pag yaad pe gop ll all nll oil F258 agitls Gata -A hdl gale Fels pppas -B ost pls -C PSs hes -D ode gl ph -E B1_| Which one of the following treatments for diabetes mellitus | a,gax ge quit g Aull old GRMN CLASU gm gl reduces insulin resistance? toy A) Acarbose (Glucobay) B) Sitagliptin (Januvia) Seas A ©) Repaglinide (Novonorm) oie -B D) Exenatide (Byett) spate -C E) Pioglitazone (Actos) 238 D Lequel des traitements suivants du diabéte qui réduit la e238 -E résistance a Pinsuline? A) L'acarbose (Glucobay) B) La sitagliptine (lanuvia) ©) Le répaglinide (Novonor) D) L’exénatide (Byetta) E) Pioglitazone (Actos) 82 | Which of the following is the most common cause of death in patients whose hypertension is under control? A. Aortic dissection B. Coronary artery disease C. Left ventricular dysfunction D. Renal failure E. Stroke Lequel des énoncés suivants est la cause la plus fréquente de déceés chez les patients dont hypertension est sous controle? A La dissection aortique B. La malasie coronatienne C. une dysfonetion ventriculaize gauche D. Insuffisance rénale E, Accident cerebral ee Se BEML SSVI ll GIST Ge PAB dl nS bb J oN Gs A ilydyoel B Srl sal thy gia -C IS ayab -D Mela. E 83 ‘A Ti-year-oid girl with Goodpasture syndrome and chronic seral failure is not compliant to her dialysis sessions nor to her antibypertensive medications. Her serum ionized calcium is 5.9 mg/dl with phosphorus 6,2 mg/dl and albumin 3.6 g/dl Whici of the following bone lesions is she most likely to have? A. Osteitisfibrosa cystic B. Fibrous dysplasia C. Osteosarcoma D. Giant cell rumor E. Osteopetrosis Une fille de 17 ans avec le syndrome de Goodpasture et insuffisance rénale chronique mest pas conforme & ses séances de dialyse, ni 8 ses médicaments anti-hypertenseurs Som (aux sérique de calcium ionisé est de 5,9 mg/dl aver le phosphore de 6,2 mg/dl et de Malbumine 3,6 g / dl, Laquetle de-ces lésions osseuses est-elle plus susceptible avoir? A. ostéite fibro-kystique B. La dysplasie fibreuse C. Lostéosarcome D. tumeur a cellites géantes E. Ostéopétrose Daag Eh ih le Wage ast halal tea oe As one gS analy aN pth odes Ga a A Vy og gl Sasa Typha Dele 84 glen go bl PRD LMI On gl JV To rel Ba ys as Sg i La I! el Al ei a =A gill qual JB Ryahe Masse -C aS USI -—D ill a 84 “An adult obese male presents in respiratory distress and is rapidly intubated. No history can be obtained prior to intubation. By clinical evaluation, be isin pulmonary edema. Of the following, what etiology of bis pulmonary edema is, ‘most likely? Smoke inhalation WV heroin use Sepsis ‘An acute myocardial infarction Pancreatitis POOBE Un homme adulte obése présente une détresse respiratoite et est rapidement intubé. Aucune histoire n'a pu étre obtenue avant lintubation. A 'évéluation clinique, il est en cedéme pulmonaire. Parmi les suivants, I’étiologie la plus probable de son oedéme pulmonaire est? A. Liinhalation de fumée B. consommation dhéroine JV C. Sepsis, D. Un infacctus aigu du myocarde E, Paneréatite goal Laat Mite ye St hy pe Garay Sh IB no 5 pe ag ga oe Se Bassey ie ae cag rl el lal oe Raita 8 ISU clase Set gl Ge gl tds he ae 5 oe A (5522 sige pakiad B Ko D 5 85 ‘An 81-year-old male has type 2 diabetes mellitus, mild renal failure and moderate ischemic cardiomyopathy. His a hemoglobin Ale of 10.9% and is on maximum dosage of, slipizide. ‘Which one of the following would be the most appropriate change in this patient’s diabetcs regimen? ‘A) Add metformin (Glucophage) B) Add sitagliptin (Ianuvia) ©) Add pioglitazone (Actos) ) Initiate insulin therapy ‘Un homme de 81 ans a un diabéte de type 2, une légéxe insuffisance rénale et cardiomyopathie ischémiquemodérée. Son taux dhhémoglobine Alc est de 10,9% et il se trouve sous ie dose maximale de glipizide. Lequel des éléments suivants serait le changement le plus approprié dans le régime du diabete de ce patient ? ‘A) Ajouter Ia metformine (Glucophage) B) la sitagliptine Ajouter Januvia) ©) Ajouter la pioglitazone (Actos) D) Initierlnsulinothérapie pad Teg Sorel ge gly Ue AY oa Uy 1 Ii ae Sel y Gs 5.6 alt ake gm Js 984% YASHBAIC A BEAM LO jas Sh il on gh fale ey Sul line inate oad A gees ta -B pile gett -C cals Skea eat -D 86 ‘A 29 y.0. primigravida has sustained bypotension after ‘marked vaginal bleeding post- delivery, necessitating transfusion of 12 packed RBC units. Postpartum, she becomes sluggish, and tired, Laboratory findings include hyponatremia, hyperkalemia, and hypoglycemia. The most likely diagnosis is: Bilateral adrenal hemorrhage Pituitary necrosis Subacute thyroiditis Metastatic choriocarcinoma Insulitis moOOw> Une ferme de 29 ans, primipare est hypotendue aprés un saignement vaginal sévére aprés l'accouchement, nécessitant tune transfusion de 12 unités culots globulaires. En post- partum, elle devient lente et fatiguée. Les résultats du laboratoire comprennent une hyponattémie, une hyperkaliémie et hypoglycémie. Le diagnostic le plus probable est ‘A. Hémorragie bilatérale des surrénales B. Nécrose hypophysaire C. Thyroidite subaigue D. choriocarcinome métastatique E. insulite “a gal «AW ball ala he TA paw yal ea a adie ih Satan els OA Stany Talal eo dh elyali yy eal ell jae Ray ie anna ec an Lai gS pal In ll eigen alg bal Cae 5 SS pap gal Soe all A GSA, eas B scare ie tell Upietetthe D Set Sul ial tel ] A manual worker presented with pain, numbness ofthe right hand especially at night. The mast probable diagnosis ‘A. Hemiated cervical dise B. Carpal tunnel C. Thoracic outlet syndrome D. Tenosynovitis of the hand Un travailleur manuel se presente pour douleur et fourmillement surtout noctumes de la main droite. Le diagnostic Ie plus probable est ‘A> Une hemie discale cervicale B- Un canal carpien C- Unsyndrome du defile thoracique D- Une tenosynovite de la main ata aa al Noah el Ge Se Dale ge Daina gS laa lve Bg SOA eet gill -B puall gate -C sigs Stet) -D 88 ‘A 49-year-old woman has had increasing cold intolerance, weight gain of 4 kg, and sluggishness over the past 1wo years, Her TSH is high; her T4 is normal. A year ago, anti- thyroglobulin and anti-microsornal autoantibodies were detected at high titer. Which of the following thyroid diseases is she most likely to have?” De Quervain disease Papillary carcinoma Hashimoto thyroiditis Graves disease None of the above moOD> Une femme de 49 ans # de plus en plus une intolérance au froid, un gain de poids de 4 kp, et a lenteur au cours des deux demigres années, Se TSH est élevée, son T est normal. II ya.un an, auto-snticorps anti-thyroglobuline et anti- microsomes ont été détectés & un ttre élevé. Parma les maladies thyroidiennes suivantes estelle plus susceptible davoir? A. maladic De Quervain B. un carcinome papillaire une thyroidite d Hashimoto D. le maladie de Graves E. Aucune de ce qui précéde ai ia op aad pe Ga gis Ge 1 pe aL TSH cae a 3818 Sy yy BAS EL an hc egal TA ee Bhi Sinall Sac BBD aN, oa le yd IST ga FA yall al AVI Ge Ua ses lel rhs BLS pe A Augie be BB Spas dalctal C es D Angee yt ely VIS LE 89 ‘A 25 y.0. previously healthy woman complains of a slight headache for 3 days, then dies suddenly. At autopsy, the heart is found to be enlarged and dilated with only minimal coronary atherosclerosis and with normal cardiac valves. The most likely etiologic pathogen to have caused her death is ‘Coxsackie B virus Aspergillus albicans Streptococcus viridians group Staphylococcus aureus Streptococcus group A moomP Une ferame de 25 ans, auparavant en bonne santé, se plaint d'un léger mal de téte depuis 3 jours, puis meurt subitement, ‘A Yautopsie, le cozur est gros et dilaté avec une athéroselérose coronarienne minime et des valves catdiaques rnormales, L'agent pathogéne de sa mort le plus probable est A. Virus Coxsackie B B. Aspergillus albicans C. Streptococcus du groupe viridians . Staphylococcus aureus B, Streptacoceus du groupe A Hr Se ak cu Ay gl Ta Gas ple 2 ad a Say pata lS SY anal Jalal Gh nl A a lS ARDY GS onal Lae 5 B SSS ut ap ake Syidcall pial OS Ae pane Mo gin Ch ARES CS moow> 90 ‘A 23 y.0, woman has worsening malaise, with a malar skin rash persisting for 3 weeks. Echocardiogram reveals sinall vegetations on the mitral valve and adjacent ventricular endocardium. Her ANA is positive, The most likely diagnosis is: A. Polyarteritis nodosa B. Scleroderma C. Systemic lupus erythematosus D. Infective endocarditis E, ANCA.associated granulomatous vasculitis Une ferme agée de 23 ans a une aggravation des malaises, avec une éruption cutanée malaire persistante pendant 3 semaines. L'échocardiogramme révéle de petites vegetations sur la valve mitrale et 'endocarde ventriculaire adjacent. Son ANA est posit. Le diagnostic le plus probable est A. Polyzrterite noueuse B Sclérodermie C. Lupus Erythémateux systémique D. endocardite infeetieuse E, Vascularite granulomateuse associée & ANCA Eo SOA on la VT ee Bl Dass ail gael Ta asin ge yale cals sa ll Fal plan le byte agi gen FS pas glist ANA gph Gla Abn gil akin ha Sa as Aiden Aa 48 a SS gl CB Ba i a lle gil ANCA moow> a Which of the Following patients is most likely to bave @ breast lesion consisting of 2 sotitary wellcircumscribed 1 om firm nodule palpated in the lower outer quadrant on physical examination: ‘A. 62-year-old woman with enlarged, firm axillary lymph nodes B,_ 20-year-old man with bilateral breast calargement C. 45-year-old woman with bloody nipple discharge D. S.year-old girl with precocious puberty E. 22-year-old woman who is asymptomatic Lequel parti les patients suivants est le plus susceptible avoir une tésion du sein constitué dun nodule solitaire ferme bien circonserit de 1 cm, palpé dans le quadrant inférieur externe lexamen physique: A. Jeane ferme de 62 ans, avec ganglions lymphatiques anillaires fermes et grands B. Homme de 20 ans avec élargissement bilatéral des seins CC. Femme de 45 ans avee un écoulement sanglant du mamelon D. Fille de $ ans avee une puberté prévoce E, Femme de 22 ans, asymptomatique BIE yal A Sl Asal 9S I er J aa Oe gl ath htt ny Aaa a pe Oe Tpoanevaad ell J) glia ga JS 69) aS a ls Le £2 Uae Li -C Lata oe SHE she lige) PL we #2 -D alyel ye SY Le TY ape thd -E, Dy Cancer of the male breast is uncommon. The most fomrmon type of breast cancer in the male is: Lobular carcinoma Ductal carcinoma Sarcoma Lymphoma Metastatic carcinoma mono Le cancer du sein chez "homme est rare. Le type le plus commun de cancer du sein chez lhortime est le suivant A. Le carcinome lobulaire B. Le carcinome canalaire C. Sareome: D. Lymphiome E. Le carcinome métastatique EAE oe pate yok ng) Pda Mae gpa a alle yd st nee Sle po A tsi B usk C Lal D Ayo te OE 93 In a hospital cardiac care unit, there are three patients with different cardiac conditions: ¢ 52-year-old man with dilated cardiomyopathy, an 18-year-old girl with mitral valve prolapse, and a 30-year-old man with infective endocarditis of the mitral valve. Which of the following features do all these patients most likely share? Decreased compliance Depressed myocardial contractility Infectious etiology Mitral valve stenosis Risk of systemic thromboembolism moAw> Dans une unité de soins hospitaliers cardiaque, il y a trois patients atteints de différentes pathologies cardiaques: un homme de $2 ans avec une cardiomyopathic dilatée, une fille de 18 ans avec un prolapsus de la valve mitrale, et ux homme de 30 ans avec une lendocardite bactérienne de la valve mitrale. Laquelle des caractéristiques svivantes est commune & tous ces patients? ‘A. Diminution de la conformité B. Ditninution de la contractlité du myocarde C. Une étiologie infectieuse D. Une sténose de la valve mitrale E. Un risque systémique de la maladie thromboembolique BOE any git JSD AL ys ON op Ja He he Ge hn pin Ua jae BU sce yy ine AD a hs Ue ae Ja pl al anal gli a ate 1A SB AB AIG A ge la eT La pl AIS pla on ag il pla fod aR ow ie jal a5 “pf hi a yt i KS al Pl Bal placa gf Get hem HS pene Sapte moowb 34 Which one of the following tinea infections in children always requires systemic antifungal therapy? A) Tinea cruris B) Tinea corporis C) Tinea capitis, D) Tinea pedis E) Tinea versicolor Lequel de ces infections mycotiques (Teigne)chez les enfants nécessite toujours un traitement antifongique systémique? A) Tinea cruris B) Tinea corporis C) Tinea capitis, D) Tinea pedis E) Tinea versicolor (Cio A Ja aoe 4A Cal om gl Ta slge sph ne alae EU Maie oA ollie -B At ee -C poate -D ype tine -E 95 [A patient on antibiotics for a wound infection develops progressive anemia, initially normocytic, later macrocytic. ‘Then he develops pronounced pancytopenia. The reticulocyte count is decreased. The most likely diagnosis is A. Thalassemia B. Aplastic anemia C. Fragmentation syndrome D, Sickle cell anemia E, Paroxysmal nocturnal hemoglobinuria (PNH) Un patient sous antibiotiques pour une infection de la plaie développe progressivement une anémie, initialement normocytaire, plus tard macrocytaire. Puis il développe une pancytopénie prononcée. La numeration des rétculocytes est dimimuée, Le diagnostic le plus probable est A. Thslassémie B. Lanémie aplasique CC. Le syndrome de fragmentation D. Drépanocytose E, Hémoglobinurie paroxystique nocturne (HPN) Sa Ce le aye Sal lg ae LSS IS sie pe ssh eo JA GALL SA des 9 SY SD aS gael pp 8 BS Gag) SRS aa ya atts ee a Ga 3 eit Bsa a Ay moowd ‘96 | An 18-year-old male presents with sore throat and has a positive heterophil antibody vest for infectious ‘mononucleosis. Appropriate management includes which one of the following? ‘A) Prednisone high dose B) An antihistamine C) Amoxicillin-Clavulonate high dose for 14 days D) Strict bed rest E) Avoidance of contact sports (e.g. wresthing) Un jeune homme de 18 ans se presente avec des maux de gorge et un test positif aux anticomps hétérophiles de la ‘mononucigose infectieuse. Une prise en charge approprigecomprend lequel des suivants? A) Prednisone a forts dose B) Un antinistaminique ©) Lamoxicilline-Clavulonate forte dose pendant 14 jours D) le repos au lit strict E) Bviter les sports de contact (e.g, lutte) Diaaly Ghul gd yay Lie Vhooae da eel ln sh aga GAY 5 ylaiel Ia PED in Gl ete Saal on De ee egy Stee VE Sad GUIS dull yal Ge We ee pa oil gb ase) Ge eee) Gly Sat Aas oa A B c -D -E 7 ‘All of the following are thought to be risk factors for the development of gastric adenocarcinoma EXCEPT: Helicobacter pylori infection High salt diet Diet high in smoked foods Cigarette smoking Alcohol consumption room Tous les suivants sont considérés comme des facteurs de risque pour le développement de ladénocarcinome gastrique SAUF A. Linfection par Helicobacter pylori B. régime élevé en sel C. Alimentation riche en aliments fumés D, La cigarette E, La consommation d'alcool as Tp Spa aa Ll oe pig VOUS lac Luann yl yll heii A, ij gle ib ie ls -B Aas YL otis (lls -C Baath ys -D past eDigasl -E 98 ‘A 2-year old firefighter sustains 30% total body surface area (TBSA) burns to his thorax, face, and extremities. His ‘wounds arc treated topically with silver nitrate, Which of the following complications is associated with use of this agent ? ‘A. Hypernatremia B. Metabolic acidosis C. Hyperchloremia D. Neutropenia E, Hyponatremia ‘Un pompier age de 24 ans est atteint de brulures de 30% au niveau du thorax, de la face et des extremnites. Ses plaies sont traitees localement par les nitrates d’argent. Quelle est parmi les suivantes la complication qui est associee a cet agent Hypernatremie Acidose metabolique Hyperchloremie Neutropenie Hyponatremie moow> Tale Geet Gaile VE oper sab ae sash Sad jn le By pianos ghee 9! Ral A jas Gea ye aby Cae GLAM, Tien acl ga Hl AR cde Serbs A (oli gas B pb osSbt -C Yee vai -D pl pg2ye Gai -E 9 ‘A 4-year-0id has abdominal pain and 3+ proteinuria on a dipstick. Three days later the pain has resolved spontaneously, and a repeat urinalysis shows 2+ proteinuria, with normal findings on microscopic examination. A metabolic panel, including creatinine and total protein, is also normal Which one of the following would be most appropriate at this point? A) Renal ultrasonography B) A spot first morning urine protein/ereatinine ratio ) An antinuclear antibody and complement panel D) Referral to a nephrolowist Un enfant de 4 ans, a des douleurs abdominates et une protéinurie 3 + sur une bandelette reactive. Trois jours plus tard la douleur a dispara spontanément, et une analyse urine répétée ssontre : protéinurie & 2 + ot des résultats normaux 8 examen microscopique. Un bilan meétaboligue, y compris les protéines totals et la créatinine sont également normaux. Lequel des éléments suivants serait le plus approprié & ce stede? A) échographie rénale, B) Un examen du taux protéine/eréatinine des premiéres urines du matin C) Un panel complet anticorps et complement D) Référer chez un néphrologue aM ah ll Ge en pa F ne ile Uiplb Juss sles alas YI Cas AULA any 47 opt gaaily Shasta 41 Hay» lc al ey YS heal aad Ca PB cg! Rene OS I ely oa Pe doa Fook Ut» | pS ape ga A ahem Un City Sly Sane gant -B ALS2 5 yaa of 99) deen tne -C (Swab al -D ca 100 ‘A @-year-old man comes for follow-up after @ recent bout of ‘acute bronchitis. He reports having a productive cough for several months, He gots breathless with exertion and notes that every time he gets a cold i lingers for months. He has been sinoking for 30 years. A physical evamination is negative except for scattered rhonehi, A chest ratiagraph done 4 raonths ago showed some byperinflation and flattened diaphragm. ‘Which one of the following is the most likely diagnosis? A) Recurrent bronchitis, B) Lung cancer ©) Asthma 1D) Chronic Obstructive Pulmonary Disease E) Heart failure Un homme de 62 ans vient pour le suivi aprés un récent episode de bronchite aigué. I! déclare avoir une toux productive pendant plusieurs mois. II le souffle coupé par effort et note que chaque fois qu'il attrape ua rhume ce Un homme de 65 ans se piaint d'une dysphagie pour les aliments solides. Une gorgee de baryum montre une lésion présque obstructive iesophage thoracique proximal. Quel est le test diagnostic suivant? A. Le scanner du thorax B. La manonséirie oesophagienne C. Echographie de 'abdomen D. Endoscopie digestive haute E. Artériographie T06 | Weinen who use Tow-dose esirogen ofal contraceptives have | 5. Wibi pg gamba cu pans apy 2 50% lower risk of cancer of the ph poctges juit¥Go. Leap cast ony ad A) breast B) cervix o €) head and neck ela D) lung ee) B ovary Sailyus -C Les ferumes qui utilisent des contraceptifs deestrogéne a oD |_| faible dose par voie orale ont un risque 50% plus fable de aa E cancer ‘A) du sein B) éucol ©) dela te et du cow D) dv poumon E) de Fovaire 107 | A 65-year-old man complains of difficulty swallowing sold | uM gi piimmos Aull Veen da sag gh Mle Ee py Ah Maly Sh Sah Fl pA andl Le pall al eal gases ith eae “A eg plaka GASB 108 Initial tfeatement of hypercalcemia secondary to metastatic A) Caleitonin-selmon subcutaneously B) Paridronete disodium by intravenous infusion ‘C) Normal saline intravenously D) Furosemide intravenously E) Parathyroid hotmone extract subcutaneously Le traitement initial de lhypercalcémie secondaire & un cancer métastatique est A) La calcitonine de saumon sous-cutanée B) Le pamidronate disodique par perfusion intraveineuse C) Sérum salé physiologique voie intraveineuse D) Furosémide par voie intraveineuse ) Extrait de Vhormone parathyroidienne par voic sous- cutanée AB PI pp AAW! GB ” BaD C5 Spat h sinaS «AL B28 ey 9) al yh Ey sab -B gx rdy isc -C arenas -D so cas Gp la Gye Led -E 109 ‘Which one is the most likely cause oF 20 year old's woman dysuria with negative urine culture? A) Escberichia coli B) Chlamydia trachomatis ©) Candida albicans D) Staphylococcus saprophyticus Quel est la cause ia plus probable de la dysurie d'une femme de 20 ans avec une culture d'rine négative? ‘A) Escherichia coli B) Chlamydia trachomatis €) Candida albicans D) Staphylococcus saprophyticus ape pl Se oe ISN mall Gli Ong! Saphe Jall £39 So He Oe gil Le Ve Sasha SAY! “A Spa Bod -B val hae C Ae sil ot SD. 110 ‘Which one of the following interventions is most likely to be suceessfl in a 16 year old girl with anorexia nervosa ? ‘A) Family -based treatment B) Adoiescent-focused individual therapy C) Fluoxetine (Prozac) D) Phenelzine (Nardil) E) Desipramine (Norpramsin) Laguelle des interventions suivantes est la plus susceptible de réussir chez. une jeune fille de 16 ans scuffranc d'anotexie. mentale? A) traitement en milieu femitial B) thérepic¢ individuelle axée sur !'adolescent C) La fluoxetine (Prozac) D) phenelzine (Nardil) B) Désipramine (Norpramin) oe se Gs BM ga IS CIA on gl f ppm Maa Olah Oa Gilad Ube 1 ‘ile Ayes A, HS g nS Ae glee -B CiBhS ys - Lienayme de conversion est augmenté est dans A Ta tiborenlose B. La lépre C. La sarcoidese D. Le lymphome E, Toutes ces réponses sont justes PSM Gaia as A CULO! dalels ta aha Lapa Banee del 4 pdI US moowd Tia | A Ss.year-old woman who has hypertension, hyperlipidemia, | a dun guj quglstie © ape Ap and osteoarthtis ofthe knees develops acute goutandis | 4. et enea wae found to have byperuricemie. Discontinuation of which one | “> UE WES SoS ok gabe Joa ean ofthe following medications roay improve her OHS ce SACS Oy gee BALL ly hyperuricemia? Tyo ane on Say OS ASL ce fiyoclootiende aoe A satan ” C) Metoprotol 28403 -B D) Simvastatin aie -C B) Acetaminophen | Une femme de 55 ans quia de Ihypertension, de hyperlipidemic et une ostéoarthrite des genoux, développe la goutte aigué et se trouve & avoir une hyperuricemie. | Larret de quel des médicaments suivants peut améliorer son biyperuricémic? A) hydrochlerothiazide | |B) tosartan || c)Lemétoprotot | | Dpto simvastatine F) Lacétaminophene T13 Laboratory findings in Hemolytie-Uremie Syndrome include all the foliowing except: ‘A. elevated Haptoglobin B. elevated Reticulocytes C elevated Bilirubin D. Coombs’ test negative (except S. Pneumoniee) E. elevated Urea + elevated Creatinine Les résultats de Jaboratoire dans le syndrome hémolytique et ‘urémique incluent tout ce qui suit, sauf: ‘A. elevation de I” Haptoglobine B. elevation ce Réticulocytes C, elevation ce Bilirubine D. Test de Coombs négatif (a Vexception de S. pneumoniae) E, elevation d'Urée + elevation de Créatinine Tl Date Die At pa gia gu Nae Le gly Le Saas al cask Ha CA, akties -B oes al Sebald 6 eb) ale eS aad Ge a S EN + Uy E ly 14 Which one ofthe following is most commonly implicated in interstitial nephritis? A) NSAIDS B) ACE inhibitors © Diureties D) Corticosteroids B) Antibiotics Leque! des medicaments suivants, est le plus souvent impliqué dans la néphrite interstitielte? A) Les AINS (antiinflamatives non stéroidiens) B) inbibiteurs de ECA (enzyme de conversion de | rangiotensine) C) Les divsttiques D) Les corticostéroides E) les antibiotiques Teil AU Tp SEE oeyl types Rie ye olga 8, AGL pate 3 pas Ale -B. casa C Ais yf S558 -D ope Cla -E Ts ‘A painful thrombosed extemal hemorrhoid diegnosed within the first 24 hours after occurrence is ideally treated by A) appropriate antibiotics B) office banding ©) office eryotherapy 1D) thrombectomy under local anesthesia ) total hemorthoidectomy ‘Une hémorroide externe douloureuse thrombosée diagnostiquée dans les 24 premiéres heures apres apparition est idéalement traitée par A) des antibiotiques appropriés, B) ligature dans votre clinique ©) Cryothérapie dans votre clinique D) thromabectomie sous ancsthésie locale E) hémorroidectomie totale PRs aie gS ga A all PEA DO! Of igen Gp Role TE Jyh DE a | ting alias “A ell gta 3h -B. pl gb 2a inden -C qe pth Cas $l Quail -D JAS yyy Juat -E, Té ‘An esymptomatie 35-year-old woman has a family history of bypothyroidism and a TSH level of 7.6 uU/mL (N0.4-5.1) and a fee T4 level within the normal range. Which one of the following is most likely diagnosis? A) A euthyroid state B) Primary hyperthyroidism ©) Secondary hyperthyroidism D) Subclinical hypothyroidism E) Overt hypotayroidism Une ferme de 35 ans, asymptomatique, « des antécédents familiaux é'hypothyroidie et a un taux de TSH de 7,6 ul! mL (N 0,4 4 5,1) et un taux de TA libre dans les limites normales. Lequel des énoneés suivants est le diagnostic le plus probable? ‘A) Un état evthyroidien B) Lhyperthyroidie primaire ©) Vhyperthytoidie secondaire D) Ihypothyroidie infractinique E) hypothytoldie patente Toe lgals cal elon SY Ula To pe Aad | gS VeeTSH 5 Gi) Mes Ga A te Ga guia sali TH 9 (esr gas) deans $s SS Gal St on gl al BBall ge Pedy bibs pot gp habs otra on Gp bits vai Geely G9 LL et -E c-9 117 A #year-old man with marked ascites 1s being Weated for hepatic encephalopathy secondary to alcoholic cirrhosis, ‘What is the most likely acid-base abnormality found in this patient? ‘A. Normal anion-gap metabolic acidosis B. Metabolic alkalosis C. Anion-gap metabolic acidosis D. Respiratory alkalosis B. Respiratory acidosis Un hioname age de 42 ans avec une ascite marquee est trite pour une encephalopathie hepatique secondaire a une cirrhose alcoolique. Quelle est I'anomalie acide base la plus probablement rencontree chez ce patient ‘Acidose metabolique avec wa trou anionique normal Alealose metabolique Acidose metabolique avec un trou anionique . Alealose respiratoire Accidose respiratoire eoop> ie ay Bale tin ase Vo pe ny Le gl GS at gS gol IE ce He git el Gant F5 sll— peen anal ie Re JL A a gM einen A ay G8 B pe yo 1 ge yl Gens C Aauits G8 .D wt yuan E ng ironic use of Omeprazole is associated with which one oF the following? A) A decreased rate of hip Fracture B) Decreased vitamin B12 absorption ©) A reduced likelihood of pneumonia D) A reduced likelihood of Clostridium difficile colitis E) An increased likelitiood of iron deficiency anemia utilisation chronique de loméprazole est associ avec lequel des suivants? ‘A) Une ditnimution du taux de fracture de la hanche B) Dimimution de labsorption de vitamine B12 C) Une probabilité réduite d'une pneumonie D) une probabilité réduite de colite & Clostridium difficile E) Une probabitité accrue de 'anémie ferriprive oD he BL Da MY Oe ll Dena By Sp oad A, VYB cat Goleta polis) -B Mss Many Decal alla! -C All Staal Se Qed alist -D cone] spel Soe ell te Uae Jase) 5 119 ‘A 60-year-old male has & 20-year history of hypertension arid suffered a small lacunar stroke 10 years ago with minor residual weakness in his right upper extremity. His blood pressure is 170/98 mm Hg. In addition to lifestyle modifications, which one of the following is the most appropriate treatment for his bypertension? A) An angiotensin receptor blocker B) A beta-blocker ©) A calcium channel blocker D)A thiazide diuretitiACE inhibitor combination ) No medication ‘Un homme de 60 ans a une histoire de 20 ans é’aypertension et a subi un accident vasculaire oérébral lacunaire minirae il ya 10 ans avec une faiblesse résiduelle mincure dans son ‘member supérieure droit. Sa tension artérietle est 170/98 mm Hg. En plus des modifications de style de vie, des suivants uell est le traitement le plus approprié pour son hypertension? A) Un antagoniste des récepteurs de I'angiotensine B) Un béta-bioquant C) Un inhibitevs du canal caleique D) Une combinaison de diurétique thiazidique / inhibiteur de enzyme de conversion de l'angiotensine ) Aucun medicament By Oe YBa pia Vs oe day Vs Satake Spe ley els Bale Ge le GID Se gina: Ss HE EBS fate LAY Gib pe WAV aie cal S891 gall ya A a gt Ayn bc pt ein able ont ya) Mele A, Gel -B cols clgs le -C enV 5 ak al gan S86 GS Ls al iypol chet aie EB 720 ‘A d8-year-old day-care presents with persistent cold, It began with a runny nose, malaise, and a slight temperature ‘elevation up to 37.8°C. She notes that after 2 weeks she is now experiencing “coughing fits,” which are sometimes so severe that she vomits. On examination you note excessive lacrimation and conjunctivet injection. Her lungs are clear. ‘Which one of the following is the most likely diagnosis? A) Pertussis B) Rhinovirus infection C) Nonasthinatic eosinophilic bronchitis, 1D) Congh-variant asthma E) Gastroesophageal reflux Une travailleuse de garderie, agee de 38 ans, se présente aver un rhume persistant, Qui a commencé avec un écoulement nasal, un malaise, et une légére élévation de la température jusqu'é 37.8 ° C, Elle note que, aprés 2 semaines, elle a aujourd'hui des quintes de toux, qui sont parfois si graves, quelle vomit, Lors de l'examen vous notez un larmoiement oxcessif et des conjonetives injectées. S¢s poumons sont lairs Lequel des énoncés suivants est le diagnostic le plus, probable? A) la coqueluche B) Infection par le rhinovirus ) Eosinophilique non asthmatique bronchite D) Asthme & variante de toux E) Reflux gastro-esophagien eS Jad ae i Send le FA pe fled Be sarhages CUialy Ons sydd Dna tay pain lS MIA cae gud ey Ay Call AOTALA Sat OSS) ny BN Gf la Cha gl Oa ll BSE oe OI Gaal ale oa ASI Aa al Cae I on gh A Lele al, Hass Ml Galt -A Aa doy all JIB RIV gels ne gl -C ay ialeaia 9) -D te gee EG! -E oa A27-year-old woman presents with bloody diarmhea and abdominal cramping secondary to Escherichia coli 0:157 Which one of the following is trae about the management of this patient? {A) Levofloxacin prophylaxis should be preseribed to close contacts 1B) Monitor patient liver enzymes ©) Patient is at risk of hemolytic uremic syndrome D) Infection was most likely acquired from well cooked bamburger ‘Une femme de 27 ans se présente avec une diarthée sanguilonante et des crampes abdominales secondaire & V'Escherichia coli O: 157, Lequel des énoneés suivants est vrai propos de fa prise en charge de ce patient? A) La lévofloxacine en prophylaxie doit étre preserite pour Jes gens qui sont en contact intime B) Suneiller les enzymes hépatiques de la patiente. C) La patiente est & risque de syndrome hémolytique et uurémique Dj Linfeetion a probablement été acquise a partir d'un hamburger bien cuit a ate Dead oe gh le pe ds Ge Gh NVRO Agi LS eV ey gt Ty pala Cal A conn gl ee cal gy GS jd hagas -A aS ta Has 48 -B pS ARS Ge Spas Saag Lael = Asll Pa ie YN Ne Samal al om lle -D toh 122 [A young patient presented to the emergency room with head | ial Ga gly ayes Jiphll ad gl) GLE am injury. He is conscious with temporal bone fracture on the rn S ‘hacia ad right, Quickly he deteriorated and became comatose with theled 2S here enh sgh A ND Fight mydriasis, The most probable diagnosis is aay he phn agi and gh ga A> Acute subdural hematome: Dein all park all! intl Abaall Qs Oe Age gh B. Extadural hematoria > C- Brain contusion da blag wey -A De Intracerebral hematoma ACB B ‘Un jeune homme se présente aux urgences pour trauma lees -C crdnien il est conscient, i! a une fracture temporale droite. Io gels esa3 py) -D. Rapidement il se détériore avec coma et mydriase droite. Le ee gee diagnostic le plus probable A. Hematome sous dural aigu B. Hematome extradural C. Contusion cerebrale D. Hemoatome intracerebral 123 | Which one of the following is most characteristic of eo A, inal econ ll Gag] patellofemoral pain symirome in adolescent females? ‘A) Posterior knee pain B) Pain exacerbated by walking on a flat surface C) Inadequate hip abductor strengta D) A high rate of surgical intervention Lequel des énoneés suivants est le plus caractéristique du syndrome fémoro-patellaire chez les adolescentes? A) une douleur postérieure du genou BB) une douleur exacerbée par la marche sur une surface plane ©) force inadéquate de l'abducteur de la hanche 1D) Un taux élevé de intervention chirurgicale i yl) sa ae gil AS ASS OA Jase gla gle gil 36 yy) -B lh Sid alg elie 9 4, Ge yalyp Clad Gall SS -D. 124 | Which of the following statement about byperparathyroidism | bub gi Say aan 9 SAD GI fall Gm gh is false? teal ‘A. Serum caleium levels are raised a B. Serum parathormone levels are raised hap il oy alo C. Bone density is low Sal eann chin es B D. Urinary calcium levels are raised at E. Intestinal absorption of caleium is decreased pet ais C Quelle affirmation est fausse dans I"hyperpara-thyroidie seis panicles 58D primaire poly pal palais) yas E A. Le calcium sérique est haut B. La parathormone sérique est heute C. La densité osseuse est basse D. Le caleium urinaire est élevé E, L’absorption intestinal du calcium est diminuée 125 [A SS.yearold woman requires @ Taparotomy for ovarian | ik jaca gly aclu Wie 06 Wye sll cancer. She has 2 history of stable angina. Which of the ss aly ina yas following characteristics is most likely to predict | 01 9) Ae Ghee nse be ant anne perioperative ischemic events during her noncardiac surgery | SY‘ +ur ol Lye 8 JV ga Sith ital yl ’ Pg Igglee JUS dal pals Deyn ST A. Angina on A B. wer. than three premafure ventricular contractions | Big yuk at OG SI B. C. Dygpmeton exertion | st cal ee -C D. Tricuspid regurgitation wk GD. E. Herage use E Une femme agee de 55 ans necessite une leparotomie pour un cancer ovarien. Elle a une histoire d'une angor stable. Quelle est parmi les caracteristiques suivantes celle qui va predire des evenements ischemiques lors de sa chirurgie non cardiague ‘Angor Plus que trois extrasystoles ventriculaires par minute Dyspnee a effort Regurgitation tricuspidienne Son age moOw> tt 126 | Signs and symptoms associated with a hydatidiform mole A Saal ga! es BS gl Gaal je Vy SLE! Gy! include all of the following EXCEPT be UghledS eas | A: First—trimester bleeding : B- A uterus larger than the expected gestational age esd Oe IN alll gf 53 A | Ce Hypothyroidism SSI el ne on Seay -B D- Preeclampsia at 14 weeks gestation Ge oats -C E- Nausea and vomiting ote Em gicesaet -D bot | Les signes et symptmes associs & une méle hydatiforme aayyfe -E comprennent tous les éléments suivants SAUF: A- Saignement du premier trimestre B- Un utérus plus grand que l'age gestationnel attend CC Linypothyroidie D- La pré-éclampsie & 14 semaines de gestation, E- Les nansées et les vomissements 127] A 50 year old male presents with weakness of the lower] Guha) bck ys KE Leo oye Jey extremities. On clinical assessment, among other findings, he 2 Skit Tost the sense of positon of the geat toe. The most Hkely | “SA oatt Sth 86 Ona ont dia oh lesion is atthe level of the ASI ALA Gl gl Hamng 8 gen) A. Posterior cord Sie le Ea gi B. Anterior cord salt A |. Cauda equina eel JB | D. LS root plea C Un patient de 50 ans se présente pour faiblesse ax membres L545 D infétieurs. A examen on retrouve entre autre, une perte de la sensibilité de 1a position du gros ortel. Son probléme se situe probablement au niveau ‘A. Des cordons postérieurs B. Des cordons antérieurs C. Dela queue de cheval D. Dela racine L5 128 | You receive at your consultation & 75 year-old woman who MIT Sedillo SEU Vee dd is vomiting since 3 days with signs of debyctration. His, eh diene cs ea At) arterial blood gases showed: rh Noe 8 AS pH 7.45, pCO2 = 45 mm Hg; Bicarbonates = 35 mmol /1. 72 = 9 SS) pa £28PCOD V4 PHL This isa 2m Dim he ‘A: Respiratory alkalosis eit A B- Respiratory acidosis Ss oyanes C- Metabolic alkalosis vitones B D- Metabolic acidosis oe -C E. Mixed alkalosis elie es =D. Aba gb -E Vous recevea 4 la consultation une femme de 75 ans pour des vorissements depuis 3 jours avec des signes de déshydratation. Ses gaz du sang montrent: pH= 7,48; pCO2= 45 mm Hg;Bicarbonates= 35 romoles/.Ts'agit dune: Ac Alcalose respiratoire B. Acidose respiratoire C- Alcalose métabolique D- Acidose métabolique E> Alcalose mixte 12 | The following is NOT true about Tinee versicolor: A. Itis caused by Malassezia furfur B. Itis maculopapular C. Tt may present as hypopigmented lesions D. Itinvolves the chest and back most commonly E. It occurs most commonly in cold climates ‘A propos du Tinea versicolor, laquelle des propositions suivantes est fausse A. Ble est causée par Malassezia furfur B. L'ertheine est macuio-papuleux C. peut se présenter comme des lésions hypopigmentées D. Matteint la poitrne et le dos le plus couramment E. I survient le plus souvent dens les climats Froids VAG yal Gala gan ep Glog ARAN jp papell eal gh Ete a ya Ela dans iil St gle 4 oI Se ABMs pal gle hE SS ya SoU elalt goWe Gans moow> 130 [A 28-year-old male has @ midshaf tibial stress fracture. ‘Altneugh he can walk without pain, he eannot run without pain. The most appropriate treatment at this point includes which one of the following? A) A short leg walking cast B) A non-weight-bearig short leg cast ©) A non-weight-bearing long leg cast, D) Am air stirrup leg brace (Aircast) E) Low-intensity ultrasonic pulse therapy Un homme de 28 ans a une fracture de stress de la diaphyse tibjale. Bien qu'il ne puisse courir sans douleur, il peut marcher sans douleur. Le traitement le plus approprié a ce stade comprend: A) Platre court de la jambe 1B) Platre court de la jambe sans port de poids C) Platte long de la jambe sans port de poids 1D) Une attelle (Aircast) E) la thérapie dimputsions de fait tensité par ultrasons Gedy pgial pS lle Wace day Yell ose mae phe Oe ot pal 08k AED ISI Sqllaal of D Us Ue JI Ses 1 gl ge gl geet Og iY a ll peel oA Se dees gua pls -B Up dans Y dah hs ie Su -D BD Ai Spall Ed om Ga -E YI [The most accurate and definitive documentation of the fetal life is made by: | A> Fetoscope B. Leopold maneuver C- Bescanner D- Real-time scanner E- Doppler ultrasound La documentation le plus précise et définitive de la vie fectale est faite par: A> Un fetoscope B- manoeuvre de Léopold C- Bescanner D- scanner en temps réel E- Doppler a ullrasons Gain Sal Gisy ASI ped le Jeans OS eal il fla “A gliadin B wee ghee -C geod yume gi oe -D Spal Gai she -E 132 | In case of acute headache presenting to the emergency room. ‘The first thing to do | A. CTBrain B. Brain MRI | C. Assessment for nuechal rigidity D. Fundoscopy ‘Devant une cephaige aigue a urgence le premier geste a faire CT. scan corebral IRM cerebral Recherche de raideur de auque Fond veil gow a de It oss nS fal cle tal al Ebatl gaye ltl eet “A Ell palin gato goat -B Gada ge CoC Ou £8 is etya) -D For normal term infants, the recommended current practice is to introduce solid foods into the diet at what age? A) 2-4 weeks B)2-3 months C) 4-6 months D) 7-9 months E) I year Chez les nourrissons a terme, la pratique recommandée actuelle est dintroduire des aliments solides dans la diete & quel age? ‘A) 2-4 semaines B) 2-3 mois C) 4-6 mois D) 7-9 mois E) lan Te Aaa NG Am a tpl HOLA weiter -B eit vel -D aud we -E 14 Marked eye pain on eye movement but without redness suggests that pain is caused by AA) an intracranial process B) an ocular condition C)a retinal problem D) an orbital problem E) an optic nerve problem Une douleur oculaire marquée au mouvement d'un ocil, mais sans rougeur suggére que la douleur est causée par ‘A) un processus intracrénien B) une affection oculaire ) un probleme de rétine D) un probleme orbitaire ) un probléme du nerf optique Gohan aad ae BSL Ga a cy iy se all all sil sb ae A Late B Asch hile -C sign Mee -D pal al fe -E 138 Breastfeeding # full-term, healthy infant is contraindicated ‘when which one of the following maternal conditions is present? ‘A) Chronic hepatitis B infection B) Seropositive cytomegalovirus carrier state C) Current tobacco smoking D) Active Herpes simplex viral lesions on the breasts E) Undifferentiated fever Allsiter un nourrisson né a term et en bonne santé est contre- indiquée lorsque l'une des conditions suivantes est présente ber la mere. ‘A) une hépatite B chronique B) Etat Séropositif cytomégalovirus C) le tabagisme actuel 1D) Herpes simplex actif avee lésions virales sur les seins E) une fievre indifférenciée FTL GG A LSS teu “A LDA psa Gall yal lea els -B, ish oss poll le Mats Saya Cul ogy -D tie -E Oe gah ol 136 ‘Among the following conditions, which one is a haemaglobinopathie? A. G6PD deficiency B. Von Willebrand disease €.Spherocytosis C D. Sickle cell disease E, None of the above Parmi les conditions suivantes, laquelle est une hemaglobinopathie? A. Le Déficit en GOPD B. La Maladie de Willebrand C. La Sphérocytose C D. La Drépanoeytose E. Toutes les réponses sont fausses | AS PDE!» Ge yl AGEN Gas : GOPD 3 -A Slylyosld ed -B CH peal SEIS -C gata gil els -D fae ye el Gp IS é B D 137 | All of the following statements regarding eoute tonsillitis ate | Asoo Sail qui) li cayay aay Aa Ca fall GS tre excep: AA Can be caused by viruses as well as bacteria ak B, Enlargement of the tonsil with erythema and pus aloog eel stan eal Ae oS ul uses “A, with high fever isa definite confirmation of bacterial tee C sireplococcs cater state shouldbe vested only in Daag Neue ipeiicieumenees eran ge 9 ae D. Maybe complicated by otitis media and sinusitis. BAS ON py Apa! Ca Sall Salat ts gt -C ibs clashed Touts les propositions suivantes concernant amygéalite Ae een yen ay a aigut sont vate, Sut nha eS A. Peut étre causce par des virus ainsi que les bactéies Sa al ely alae B. Le gonflement de Tamygdale avec eryineme et présence de pus et une forte fiévre confirment une infection bactériene, C. Les porteurs du Streptocoque ne doivent éie traités que dans des ciroonsances speciale, DePeut se compliquer d'une otte moyenne et d'une sinsie 138 | Adreval incidentaloma is usually a Pheochromocytoma Hyperaldosteronisme adenoma A cortical cancer ‘Non secreting adenoma None of the above moOB> incidentalome surrénalien est habituellement ‘Un pheochromocytome Adénome avec hyperaldostéronisme Cancer de Ia zone corticale Adénome nonséerétant Aucun des précédenis, moowE Foe A ed DEB ey a eos SaaS A oe GE pas 5 Be Je SS Hd Ramee yb el aM IS moow> T39 | The occurrence of menstniation a dependent OAD Del SG gaa Bay | nat ee tie BCT eettnte B- An endometrium responsive to sex steroids Sesion gh ier Alas -B C- Gonadotropins wiley D. Patent fallopian mbes ati a basis -D E- Ovacian steroidal hormones Sakyaed yonallchiyaye -E Llapparition de la menstruation est dépendante de tous les facteurs suivants SAUF: [A- Hormone hypothalamique libérantes {Un endoméire sensible aux stéroides sexuels Gonadotrophines D-Trompes de Fallope perméables Hormones ovariennes steriiennes a0 | Indications fora cesarean section include all ofthe Following | wwe u yu ds ona y pase ley EXCEPT A- Previous cesarean section sat B Failed forceps delivery te kpatiie “A C- Fetal distress dads 59h SB D- Cervical cerelage tue tie -C E- Cord prolapsed pa desks -D | dag Les indications pour une césarienne comprennent tov les siden E suivants SAUF: A- Une eésarieane précédente B- Echee de I'accouchement par forceps C- Détcesse foetale D- Cerolage du eo! utérin E- Providence éu cordon M147 | Defects in corpus luteum function may be demonstrated by peenll Rigas gd So pall 486 gle Seana Gi Say all the following EXCEPT : ee Wyble fant ‘A- Basal body temperature seth tap A | B. Endometrial biopsy and histologic dating Ant ema Le ba C Measurement of sera progesterone levels pestle B D. Determination ofthe length ofthe luteal phase pet sian pl hasan ob -C E- Measurement of serum estrogen levels Gabel gl tyes -D N opm ase ld J Des défauts dans la fonction du corps jaune peut etre Reh owas ce is E démontzée par toutes les suivantes, SAUF: | | A.La température basale du comps B.La biopsie de Yendoraéne et datation histologique CLa mesure des niveanx sériques de progestérone D.Détermination de le longueur de la phase lutésle E-Mesure des niveaux doestrogéne sériques 142 ‘When prescribing an inhaled corticosteroid for control of asthma, the risk of oral candidiasis can be decreased by ‘A) rinsing the mouth after each administration B) limiting use of the inhaled corticosteroid to once daily C) adding nasal fluticasone propionate, D) adding montelukast ) adding salmeterol Lors de le prescription d'un corticostéroide inhalé pour le controle de l'asthme, le risque de candidose orale peut étre imoinuée par A) le ringage de la bouche aprés chaque administration 'B) limitation de utilisation de la corticothérapie inhalée & tune fois pat jour C) Faddition de futicasone nasale D) Iaddition de montelukast B) laddition de salmétérol ale hed RE a SSS ey ae Ep a cae Tay i Sy Sh thaty te yp Soe gilda “A PSY ye 6a I Qo Oa sa pall gi talstd BY ob ce iy 2s ASE Hl cfg Ll Jy Sue tt Tas ‘A SS-year-old woman presents with generalized weakness and a history of poorly controlled hypertension despite use of full dose of hydrochlorothiazide, lisinopril, amlodipine and doxazosin Her blood pressure is 165/105 mm Hg, with grade 2 retinal ‘changes and a serum potassium level of 3.1 mmol/L (N 3.5. 5.5). What is the most likely diagnosis in this patient? A) Chronic renal failure B) Seep apnea CC) Hyperaldosteronism D) Hyperthyroidisna E) Renal tubular acidosis ‘Une femme de 55 ans presente avec une faiblesse généralisée et des antévédents d'bypertension mal contrOlée malgré utilisation de la dose complete de l'aydrochlorothiazide, le lisinopril, 'amlodipine et la doxazosine.Sa pression artérielle est 165/105 mm Hg.elle a de modifications grade 2 de la rétine et un niveau de potassium sérique de 3,1 mmol ! L (N 3.5855). (Quel est le diagnostic le plus probable chez ce patient? A) Insuffisance rénale chronique B) L'apnée du sommeil (C) Hyperaldostéronisme D) Lhypertyroidi B) L'acidose tubulaise rénale Ea le de Oe st Ute 22 UA ae Lal AUIS de ym adil Gn gt le phe ye hte casi babies ta Ot NSEC NON? pl ake ge RK ay pine pial gn 5 sieny VA Gn AE A Cis CARS 9h Le (210-F 4 grata) Spe lla FH Some yl Nig Ga SSK GIS apes 8 cal eth) -B cs suayh ed -C gal bishs -D 8 gas aes EB 144 In chronie subdural hematoma, ene is wrong ‘A. more frequent at extreme ages B. can mimic any neurological disease C. increase by osmosis D. increase by bleeding between the membranes | Un hematome sous dural chronique (une est fausse) A. plus ftequent aux ages extremes B. peut mimer n'importe quelle maladie neurologique CC. grandit par osimose D. grandit par saignement entre les membranes Tsar yh aye pl i in ype pall aT we “A B Cc -D Sl Jee ys Lily St pene a gl AD Sa eat dad aly 2291 oy a5 290 9 a Ta5 ‘A 67 year-old, is treated for diabetes, hypertension and dyslipidemia. He regularly consumes 4-5 drinks per da Since July 2008, he became depressed after the death of his, only son; he cats little and merely a piece of chocolate, vegetables and potatoes, Tonight, he presented 10 the emergency room with diplopia, disorientation and nystagmus. The rectal temperature is 37 °C; his BP 120/80 mmHg and bis pulse 85/mn, What is your next step? ALIV hypertonic dextrose B-Cerebral CT scan (C-Lummbar Puncrare Denjection of 100mg of thiamine IV E.Give him a prescription of multivitamin 3 Mr G.B., agé de 67 ans , est diabétique, bypertendu et dyslipidémique sous traitement. I! consomme reguliérement 445 verres dlalcool par jour. Depuis Juillet 2008, il est devenu dépressif suite au décts de son fils unique; il mange peu et se contente d'un morceau de chocolat, de legumes et de pommes de terre cuits. Ce soir, aux urgences, il présente vune diplopie, une désorientation et un nystagraus. Sa température rectale est a 37°C; sa T.A a 120/80 mmHg et son pouls a 85/mn, Quelle ext votre CAT ? ‘A. Glucosé hypertonique 1V B- Scanner cérébral et fond dxil C- Ponetion lombaite D- Injecter 100mg de thiamine en IV E- Lui donner une ordonnance de multivitamines B Ta EU) ug SD elige gla le Wee Un AS HS Ot sy BE A ll gt HI a Di ye Ee ae TAA 5g Se pl AAD pons Uy hand OY 9H Ge US Aa eles On yl Ay ees LANE ol eA) pal aes OPV Ra yt om 33 lg Le YAS eal 5 Usury Ae gle bys OA tl gages glia gue -B ebbdy -C Gayonisge te oie -D Die Boley -E TAS [ We can find osieobastic bone metastasis Wal exmapr one pathology A. Prostate adenocarcinoma B. Pager disease C. Over absorption of Fluoride D. Breast cancer E. Bladder carcinoma ‘On retrouve des lesions osseuses métastatiques ostéoblastiques dans toutes ces affirmations sauf une ‘A. Adenocarcinome de fa prostate B. Maladie de Paget CC. Absomption excessive de fore D. Cancer du sein E. Cancer ée la vessie ea HWS Aa LS le heya A | Sebel B ay pill 21) poled Cc elghe — D Walden EB Which of the following is not an indicatton for adenoidectomy alone? ‘A. Chronic nasal infection (chronic edenoiditis) B. Chronic sinus infections that have failed medical management C. Recurrent bouts of acute otitis media . Recurrent otorthea in children with tympanostomy tubes E, Recurrent pharyngotonsillis Lequel des énoncés suivants n'est pas une indication pour une adénoidectomic seule? A. infection nasale chronique (edéncidte chronique) BB Infections chroniques du sinus qui n'ont pas 6x6 prise en charge médicalemen C- Episodes récurtents d’otite moyenne aigué D. otombée récidivante chez les enfants avec des rubes de tympanostomie E- pharyngite récurrente Thi ile Gated ye ag YI Gn gl (od le esl) Ga gil OS) WA Aaya cin cts Ge je aes LB tall eg SV eal Ob Sie Chane sala pebag sel GUM! ae 9 Se sl SD ab Ob M pall ell -E 148 ‘All of the following procedures are appropriate for ‘evaluation of the endometrial cavity EXCEPT: A> Laparascopy B- Endometrial biopsy C- Hysteroscopy De Endometrial culture Hysterograpby Toutes les procédures suivantes sont appropriées pour V'évaluation de la cavité utérine, SAUF. Allaparoscopie Be-biogsie de 'endometre C-hystéroscopie D- culture endométriale E-hystérographie Us Aad gal pul Gale gh as gly US. ae. Obs ks -A a5 B c-61 149 [A 65-year-old man presents to the emergency department with sudden onset of left retroperitoneal pain. He is 1.75 m in tall and weighs 86.2 Kg, Physical examination reveals his blood pressure to be 85/45 mm Hg, and a pulsatile mass in the abdomen is palpated. The pathogenesis of this patient's disease is most closely related to which of the following? A. Hypertension B. Atherosclerosis C. Elastic tissue fragarentation D. Immune complex-mediated inflammation E, Vasculitis secondary to syphilis, Un homme agé de 65 ans se présente aux urgences pour une apparition brutale d'une douleur rétropéritonéale gauche. Il ‘mesure 1,75m et pese 86.2kg. L'examen physique révéle que sa tension artérielle est a 85/45mmblg et une masse pulsatile est palpée au niveau du I'zbdomen. La pathogénése de la maladie chez ce patient est étroitement relige a laquelle des suivantes = A. Hypertension B. Atheroseltrose C. Fragmentation da tissu élastique D. Inflammation par Vintermnédiaire de complexes imrmuns E, Vasculite secondaire 4 une syphilis Ts eo soy pg ILE Vee Ds Be ya gl ina AY aggplie 0 Oe Sg nell pan AB SMUT ay YOO) al LAL SN Gy 4585 ab 8A pl an Pail Ant NT ARall OF lia BO gh FA a gl ats 1 38 ‘atsallg li) -A gpuee ches B cost qel gh -C fuel de ule gat -D oetlnads at BS ile y lei -E 150 | Whick one of the following patients should be advised to take aspirin, 81 mg daily, for the primary prevention of stroke? A) A 42-year-old man with @ history of hypertension B) A 72-year-old woman with no chronic medical conditions C) Ap 81-year-old man with & history of depression D) An 87-year-old woman with a history of peptic uleer disease Lequel de ces patients suivants doivent étre avisés de prendre de l'aspirine, 81 mg par jour, pour la prévention primaire de TAVC (accident vasculaire cerebral) ? ‘A) Un homme de 42 ans avec des antéctdents hypertension B) Une femme de 72 ans sans conditions médicales chroniques ‘C) Un homme de 81 ans avec des antécédents de dépression | D) Une femme de 87 ans avec une histoire dulcére gastro | duodénal ANY ace aa I hie Ae i pin ll Oe gt FRA ANAT ay Thi pli Oe gala Whe 8 eae dy “A, Bb Le Ge GG Y Lie VY agar dl pl -B aye SASSI ye clay We AY some Jay -C yee RB Ga ylly Ue AV La ae thy! -D 131 ‘A35 yo. previously healthy woman dics suddenly and ‘unexpectedly. At autopsy, one of her cardiac valves demonstrates attenuation of the chordae tendineze, with rupture of one of the chordae. On microscopy, one of the ‘mitral leaflets shows myxomatous change. The most likely ‘cause of death is Carcinoid heart syndrome Mitral valve prolapse Rheumatic hesrt disease Infective endocarditis Acute myocardial infarction monw> Une femme de 35 ans, auparavant en bonne sanié meurt subitement et de fagon inattendue. A Tautopsie, l'une de ses valves cardiaques démontre letténuation du cordage, avec une ruptate de lune des cordes. Au microscope, lun des feuillets mitraux montre un changement myxoide. La cause ta plus probable de ia mort est ‘A. Syndrome cardiopathic carcinoide B. Prolapsus dela valve mitrale C. La cardiopathie shumatismale D. Liendocardite infectieuse E, Infarctus aigu du myocarde (5 tp tay Ga AS Ge FO pe Aad pd ade a0 ga el gh ghia ut Sy lad sta) BGK go abd UY FU S seb Aaa san) Sel seals Sha ABU aa 8 SVE tall OI bbe yay aid Age JS GB THEI De A Pil plat gs aha yi Xe! gi ABE aa 4B tne Lita moa 152 ‘A 75-year-old presents with puffiness of the face, arms and shoulders associated with a bluish to purple discoloration of the skin, In addition, he complains of dizziness, shortness of breath, and cough. He has 2 35 pack ~year history of smoking, Physical examination reveals clubbing of the fingernails, emphysematous chest and distended neck veins The pathogenesis for this patient's findings most likely results from which one of the following disorders? Primary lung cancer Pericardial effusion Selerosing mediastinitis Polyeythemia cubra vera Right ventricular failure mOOD> Un homme age de 75 ans se presente avec une boursouflure de la face, des bras et des epaules associc a une decoloration bleuatre et violacee de la peau En plus il se plaint de vertiges, essouflement et toux. Il a une histoire de tabagisme 435 paquets par an. L’examen physique revele un hippocratisme digital, un thorax emphysemateux et des veines du cou distendues. La pathogenese de ce tableau chez ce patient resultent le plus probablement de laquelle des suivantes Cancer pulmonaire primitive Epanchement pericardique Mediastinite sclerosante Polycythemie rubra vera Insuffisance ventriewaite droite moaD> ORB ag GBI On Sb) Me 8 out Uns else J G5) aot all cl go ih sie GSI Day Gah pak ny eb A SRL anil el Ge le 8 a aE Gi pe Bayh Bday gate Cll gla yaad gall of Sey A a aa Ge gh AL TR yo ale! erty awe A sonicheail -B snl Guat gal Sia pati ss -D ge apa -E. 153 | Conceming systemic lupus erythematosus (SLE), all of the | | following proposivions are true, except A. It causes 2 characteristic "butterfly rash’ B. Is more common in males C. Anti-nuclear antibodies are usually present D. Maternal SLE can cause heart block in the neonate E, Possibility of pericardial effusion |_| A propos du Lupus érythémateux disséminé (LED), toutes les propositions suivanies sont justes, a exception: ‘A.Elle provogue une "éruption cutanée én papillon» caractéristique | B. Bille est plus fréquente chez les gargons ©. Les anticorps anti-nucléaires sont habituellement présents, D, LED mternelle peut causer un bloc cardiaque du nouwveaw-né E. Possibilité d'une péricardite AG SI a IS Algal ala DL ie Linen (SA pile ad A, SU) te et SIA -B Rag ot shaat ate ns -C Daa al yl ie 4 gal bea i oS -D SBD ae i a oh Sy So pliclesl te lca -E 1154 |X @a-year-old woman Ws in the ICU Temediarly Tollowing removal of a left adrenal pheochromocytoma, Her blood pressure is 80/40mmHg, The most appropriate treatment of the patient's hypotension is which ofthe following? ‘A. Phenoxybenzamine B. IV bolus of lactated ringer solution C. Epinephrine D. Phenylephrine Une femme agee de 42 ans est admise dans l'unite des soins |_| intensifs immediatement apres l'ablation dun pheochromocytome surrenalien gauche. Sa tension arterielle | | est 80/40 ramHg. Quel est le traitement le plus approprie de | | Phypotension chez ce patient A. Phenoxybenzamine B. Solution de lactate Ringer en bolus iv C. Epinephrine D. Phenylephrine shea gold tie Taye ta) Ss ighn pM LS GS AS 99 ice Gpalisy acl Cath Gh GED pe C/A A haypall Gh a onSsit “A AS sip saa atanrrstale -B cotta) -C coal Jb -D jis ‘A 28-year-old man who was in an automobile accident is brought to the emergency department by paramedics. The patient is conscious. Clinical examination reveals no sensations below the level of the umbilicus and absent superficial and deep tendor: reflexes. Which of the following, would bean expected finding in this patient? ‘A. Hypertension and tachycardia B. Hypotension and bradycardia C. Hypertension and bradycardia D. Hypotension and tachycardia E, Normal blood pressure and normal pulse rate Un homme age de 28ans qui a eu un accident d’automobile est amene aux urgences par les secouristes. Le patient est conscient. L'examen revele V absence de sensation au- dessous de l’ombilic et une absence des reflexes superficiels et profonds. Quelle est parmi les propositions suivantes celle que vous attendez & trouver chez ce patient : hypertension et tachycardie hypotension et bradycardie hypertension et bradycardie hypotension et tachycardie tension arterielle normele et un pouls normal moAp> A pl Stal lle Waa day pani Bl gyal ing os pk Se il Sn Ca a Sit ge BB Ke GI ag) Rely yall CLAS | 1 Gayl I oe gs er E Leyden gL “A A ght dba til -B cbc shying -C ibe ey bie otis -D phe al he pie Sane geste pa baie 156 Follow-up of the management of sbydaidiform mole should routinely include all of the following EXCEPT ‘A> Human choronic gorsadotropin level determinations B.- Pelvic examinations C- Contraception D- Chest films: E- Chematherepy Suivi de fa gestion d'une méle hydatiforme devraient systématiquement inclure tus les éléments suivants SAUF. ‘Ac une détermination du taux de gonadotrophine chorionique B- examen pelvien CLs contraception D.- Radiographie des poumons E- Chimiotherapie AI SE i a goal yal a 3 Nae Le dS iy dS lta a Cl sane A apall choad -B dale -C Biyelet yest -D inser -E 157 | pour une malignite, La famille demande a propos du ‘A 49-year-old woman undergoes surgical resection of a malignancy. The family asks about the prognosis. The histopathology is available for review. For which of the following malignancies does hitologic grade best correlate with prognosis? ‘A. Lang cancer B. Melanoma C. Colonic adenocarcinoma D. Hepatocellular carcinoma E, Soft tissue sarcoma Une femme agee de 49 ans subit une resection chirurgicale pronostic. L’histopathologie est disponible. Pour Izquelle des rmalignites suivantes le grade histologique est en correlation avec le pronostic Cancer pulmonaire Melanome ‘Adenocarcinome colique Carcinome hepatocellulaire Sarcome des tissus mous moOwD> ayaa Sle gpslle pe ps ea haa of 9 ge Mite tle Sans py OSTA BSI AN Oa gh Anna se # JS go Bs) SIL all Bm -A he os pis ah spall bye sN quill Le 158 ‘The most important result of the increased number of cesarean sections being performed today is A+ Decreased matemal morbidity B. Decreased matema! mortality C- Decreased perinatal mortality D- Increased matemal mortality - Increased use of fetal monitoring, Le résultat le plus important de Taugmentation du nombre de césarionnes effectuées aujourd'hui est le suivant A> Dimivution de la morbidité maternelle B- Diroinution de {a mortalité matemnelle C- Diminution de la mortalité périnatale D~ Augmentation de la mortalité maternelle E- Augmentation de Iutilisation du monitorage foetal Sia ae eR al SVT Gl ho Bsns ll nasil 28) Gal 8 4 Gali! A, apd OLA Us ality! -B oy faa CLAS Ue Galt! -C adi ctigh Siu -D Atal Aal yal pasa) saad 139 You see a 90-year-old male with a 5-year bistory of progressive hearing loss. The most common type of hearing loss at this age affects, A) predominantly high frequencies B) predominantly mid frequencies ©) predominantly low frequencies D) all frequencies roughly the same ‘Vous examinez un homme de 90 ans avec une histoire de 5 ans de perte progressive de Taudition, Le type le plus commun de perte auditive & cet age affecte A) les fréquences élevées principalement B) les fréquences moyennes principalement C) les fr€quences basses principalement 1D) toutes les fréquences & peu prés les memes a ile ge egy le 8 eae Oa Ce a Ble 2 3 gal yh eso iin pba all st A Sits thas th OB pots SS tl 588 = eis ds -D “166 ‘Abnormalities in the cervical mucus may result from all of the following conditions EXCEPT: A- Colonization of the cervix with cytotoxic organisms B- Uterine retroversion C- Chronic infection of the cervix D- Previous electrocauterization of the cervix E> Antisperm antibodies Des anomalies de Ia glaire cervicale peuvent résulter de toutes les conditions suivantes SAUF: ‘A.Une colonisation de col de lutérus avec des organismes cytotoxiques B.Rétroversion utérine C.Liinfection chronique du col de I'utérus D. Antécédant d’Electrocautérisation du col de uterus. E.Anticorps amtispermatozoides STAD IS Ge pb A la i GOS hoe LA WBS er hpi Ol pa apy “A ices gill po J pul de bores! She BES oS Chasil falaan haat T6L Tie sya wane’ of dee Tr enerocosel aa GAS gh Hye IU Tee endocarditis in a 38 year-old man, with aitral insufficiency \ ctype leit ed and normal renal function, who is allergic to penicillin, is: seat etal ch Sala . Ae Oe A. Vancomycin + gentamycin Jia lp 9p Le pp 1 al Ae D.Claforan + Gentamycin faa ged Vancomycin Rfampcin idly + Gouda ft “A ancomycin ene E, Ciprofloxacin + Gentamycin fous -B ciliny + oped Sat -C Le traitement empirique de choix d'une endocardte i coals D entérocogue chez un jeune de 35 ans, conn allergique & la cnn + dul yaee -E péniciline, t ayant une insuffisance mitrale et une fonction rénale normale ‘A. Vancomycine + Gentamycine B. Claforan + Gentamycine C. Vancomyeine + Rifampicine D. Vancomyeine E. Ciprofloxacine + Gensamycine TG] Which ofthe following is NOT an indication to Iitate treatment for chronic lymphocytic leukemia? A. A lymphocyte count that doubles in less than 6 months A hemoglobin of Ty/dL. Severe thrombocytopenia Night sweats A WBC count of 34,000/mra? moo Lequel des énoncés suivants n'est PAS une indication pour initier le traitement de la leucémie Iymphoide chronique? ‘A. Une numération des lymphocytes qui doublent en moins de mois B. Un taux d'hémoglobine Ty/dl C. La trrombopénie severe D. Sueurs nocturnes, E. Une numération des globules blanes de 34.000 / mm3 aay CaN eal gel cn cal a al on gh Fa alg sal al edd Ge BI piel by il sls HEY ow OB anche ua C was oD Teg fers slimy cb Ss BE produce lytic bone metastases except: Breast Lung Thyroid Kidney Prostate moop> Les cancers suivants produisent des métastases osscuse lytiques sauf. Sein Poumon Thyroide Rein Prostate moom> T63 | A patient with nephrotic syndrome and nonwal creatinine Ca SI gay ASAD Ge Ged Ve clearence, presents with sudden onset of left flank pain, and : vasye a emis abrctoy eon ecg deriontan of | = A te 8 py nm Sth renal function, Ultrasound demonstrates the left kidney io be | HA! ve ssill Gul Aisas Als spall Hein larger than the right. Which of the following is the most LUG oped Sy st AeA yt gam a8 likely diagnosis? Coe AD Geb arial gn pgs oa ‘A Acute pyelonephritis ei eM B. Renal artery occlusion ‘ais ; ©. Renal vein thrombosis ARBs tne che A D. Nephrolithiasis 1380 chy abst -B E, Ruptured renal eyst eS yas ses -C QScdnee -D Un patient atient du syndrome néphrotique et ayant une fySik a5 -E clairance de la exéatinine normale, se présente avec apparition soudaine d'une douleur au flane gauche, et une hématurie, Les tests de Laboratoire montrent une <étérioration rapide de la fonction rénale. L'échographie monte le rein gauche plus grand que le droit. Lequel des énoneés suivants est le diagnostic le plus probable? ‘A. La pyelonéphrite aigué B. occlusion de arte ténale C..thrombose des veines rénales D. Néphrolitbiase E, Rupture de kyste rénal Téa] Allthe following pamary malignancies are ikely to Peal GS Sie gis I A NT Gee fae et a Gl 460 sees moons C68 | Chronic bruceliosis may cause all the following except cone: A. Atthropathy B- Osteomyelitis C- Liver granulomas: D- Splenamegaly E. Hypercalcemia La brucellose chronique peut présenter tout ce qui suit sau vane ‘A- Arthropathic B- Osteomyelite C- Granulome hépatique D- Splenomégalie E. Hypercaleémic Tet bo 8 fa yl gee Cain SBLel A 8 gl -B | aS gum oy dab! ile -D MemlS hs -E with cirthosis due to hepatitis C. what will be her most likely cause of death ? A. Liver failure B. Myocardial infarction C. Bleeding varices D. Hepatocellular carcinoma E. Renal failure Vous étes entrain de voir un patient qui a une maladie hépatique terminale avec une cirrhose due & I’hépatite C, ‘gael serait la cause la plus probable de son décés ? Insuffisance hépatique Infarctus du myocaide ‘Saignement des varices oesophagiennes Carcinome hépato-cellulaite Insuffisance rénale moap> 166 [A 40-year-old woman is found to have a3em, fim, smooth | 4c. LLGSGaue i byes | ‘mass in the upper outer quadrant ofthe breast The mass is 75 gal pA pm JOD gall gi pu T Ugh aspirated, and 10 mL of cloudy green fivid is removed. he | Sacks ad goale Ve") Ate Sis appropriate management at this time is Pp cd BD pd A. Administration of estrogen FR ok B. Cytologic evaluation of the aspirated Nid ot 5 C. Excision of the cyst mana Sal gas TB D. Observation only Sof Guctind ~C "ais 41 yoh-D “Une femme dgée de 40ans présente une masse de 3cm_ ‘erme, souple, au niveau da quadrant supéroexterne hi sein, La masse est aspire et on rette 10m d'un liquide vert trouble. Ace stade la conduit a tenirappropré serait: A. Administration d'estrogéne B. Evaluation cytologique du liquide aspire C. Excision du kyste D. Observation seulement 167 | You are seeing a patient who has end-stage liver disease. el a AY! Ga yall A Le, pal till po gal 2S le LA he at oe ep CE hg Caan 8 ASV! ll ga gl Ge gt gS LB-A fyi ne ¢Ltia' -B WE Aye -C ga gas lb. -D pK dis -E (168 The presence of red blood cell casts in the urinary sediment is suggestive of: Acute hemorthagic cystitis Glomervloneptiitis, Adenovirus cystitis Acute prostatitis Pyelonephritis mone La présence de cylindres hématiques dans le sédiment urinaire sugeére Cystite hémorthagique Gloménulonephrte Cystite a Adénovirus Prostate aigue Pyélonéphrie moom> CH Gd Te LS as he a Ue algal “A AS hes yall -B PRE ees ah athe ll “C she ng ys Ag -D FAD yl yl -E 169 ‘Quelle est I'&iologie la plus probable & évoquer devant de volumincuses adénopathies bilatérales blaires et médiastinales asympromatiques associées & une conservation de I’état général et & un erytheme noueux. chez une femme de 25 ans? ‘A. Tuberoulose B. Sarcoidose C. Lymphome nen hodekinien D. Maladie de Hodgkin E. Maladie virate ‘What is the most likely diagnosis in front of ‘itetcral asyrmptomatic hilar and mediastinal voluminous adenopathies associated with good general state and erythema nodosum in a 25 y.o. female? Tuberculosis Sarcoidosis Non-Hodgkin Lymphoma Hodgkin’s disease Vira illness Ponp> PRESS ST ae ao SS gate 8b NE] ala It pune Be Bike ping tanp Hite Mla Ge Gi fa pe F Ue ¥2 La ye al al eb els A aS B OSes a Vuasial Seige ols -D eso ae E i70 “The mainstay of treatment of infectious diarrhea is Antibiotics Fluid and electrolyte replacement Anti-diarrbeal agents Dict advice. Antispyreties monw> Le traitement standard d'une diarrhée infectieuse est Antibiotiques ‘Remplacement liquidien et electrolyte Agents anti-diaréiques Conseil digtétique Anti-pyrétique moowP Te GRD Teall Sollee gi all OS aad Slalad «A SH Ss hee Gay -B Dal Balin dalye C ihe clas D sulga Ghaits E 0 Tn a patient with a sudden onsct of dyspnea, which one of the following makes a pulmonary embolus mote likely? A) Fever >38.0°C B) Chest pain © Orthopnea D) Wheezes E) Rhenchi ‘Chez un patient avec une apparition soudaine d’une dyspnée, lequel des suivants fait une embolie pulmonaire soit plus probable? A) Fievre > 38,0°C B) Douleur @ la poitrine ©) Orthopnee D) sibilantes ) rales 92 GIS Gl aS OB Gyn td ae Gia Se Spb rclaatl ol ETI Ga lel loa A goseal -B eet ath base 23D bihe -E 172 ‘A 2-year-old child stumbles, but his mother Keeps him from falling by pulling wp om his right hane. An hour later the child refuses to use his right arm and cries when his mother tries to move it ‘The most likely éiagnosis is A) dislocation of the ulna B) dislocation of the olecranon epiphysis ©) subluxation of the head of the radius 1D) subluxstion of the head of the ulna E) anterior dislocation of the humeral head Un enfant de 2 ans trébuche, mais sa mére l'empéche de tomber en tirant sur sa main droite, Une heute plus tard, ‘enfant refuse d'utiliser son bras droit et pleure quand sa mire essaie de le déplacer. Le diagnostic Ie plus probable est AA) luxation cubitale B) une luxation de Tépiphyse olécranienne ©) la subluxation de la t8te radiale 1D) une subluxation de la t2te eubitale B) une luxation antérieure de la téte humérale Blas lly Cy Aa gS aS ote ¢ a ile ele sey lel cgay AR EB Oe Us ae SG Lae Saya patio St ay ALS SSM Gea TGS as ly silk A coe -B fetta gem alk -C 2d li ge Oe -D saad yal a -E

You might also like